CLIPP Cases

अब Quizwiz के साथ अपने होमवर्क और परीक्षाओं को एस करें!

Which of the following is not a risk factor for CAD? A Hypertension B Female gender C Smoking D Diabetes

The correct answer is B. Hypertension, smoking, and diabetes (along with elevated lipid levels, obesity, and sedentary lifestyle) are modifiable risk factors for CAD. Non-modifiable risk factors include older age (men>45 and women >55), family history of CAD in a first-degree relative at a young age, and male gender.

An 11-year-old male comes to the clinic with a chief complaint of abdominal pain for three months. The pain is not associated with eating. Sometimes he feels full and nauseated, along with the pain, but then it resolves on its own. He denies diarrhea, vomiting, and bloody stools. His mother is primarily concerned because his abdominal pains cause him to miss school quite often now. ROS is otherwise negative and the only pertinent issue is his pain. When you evaluate his growth curves, he is progressing at the 60th percentile for height and weight and you do not notice a change since birth. Through a social history you ascertain that he is quite intelligent and has recently been advanced to 7th grade from 5th grade. Vital signs are within normal limits for his age and physical exam (including rectal and genital) are unremarkable. Stool sample was sent in anticipation of today's visit and was negative for occult blood. What is the most likely cause for his abdominal pain? A Functional abdominal pain B Inflammatory bowel disease C Bacterial gastroenteritis D Peptic ulcer disease E Meckel's diverticulum

A has been selected by the expert. A. Functional abdominal pain would be the most likely diagnosis in this setting at this time. History in this setting is not suggestive of any other diagnosis directly causing his abdominal pain, except a change in his social setting. For better understanding of the nature of this child's pain, it would be best to talk to him alone, without his mother present, to determine if he is having trouble adjusting to school and to assess whether he has a stable home environment. His pain is chronic, with no other symptoms (diarrhea, bloody stools, growth failure), making a functional issue most likely. B. Inflammatory bowel disease (Crohn's disease or UC) could be the cause of this child's pain, but more frequently IBD presents with growth failure, diarrhea, bloody stools, and sometimes fever. Although this could be an early presentation, the absence of arthritis, uveitis, and other constitutional symptoms, make IBD less likely. C. Choice C is incorrect because bacterial gastroenteritis is more of an acute issue and would be unlikely to persist for months. The absence of diarrhea also suggests bacterial gastroenteritis is not the best answer option. D. Choice D is incorrect because peptic ulcer disease (PUD) is relatively uncommon in children and the stool sample was negative for occult blood. PUD would be chronic and recurrent, but should also be associated with eating and would not spontaneously remit every couple of days. E. Choice E is incorrect because Meckel's diverticulum usually presents as painless blood in the stool.

A 5-year-old African-American male with sickle cell disease presents to clinic with a chief complaint of severe chest pain for the past day. His mother notes that he has been breathing quickly and that she measured his temperature this morning to be 100.5 F. Patient describes pain as an 8/10 on the faces scale. Patient is tachypneic on exam and has an oxygen saturation of 97% on room air. Chest exam reveals normal lung sounds bilaterally, and he has some reproducible tenderness over his chest wall. A chest x-ray is performed and demonstrates clear lung fields and a cardiac silhouette that is within normal limits. What is the most likely cause of the chest pain? A Acute chest syndrome B Rib infarction C Sepsis D CHF E Pneumothorax

B has been selected by the expert. A. Acute chest syndrome (ACS) is incorrect, because there is normal oxygen saturation, and normal lung fields on radiological investigation. Lung markings on chest x-ray suggestive of ACS would include pulmonary infiltrates, atelectasis, and effusion. Causes of ACS include infection, pulmonary fat embolism, or intrapulmonary sickling. Typical therapy may involve supplemental oxygen, pain management, hydration, antibiotics and possibly transfusion. B. Rib infarction is correct because of the chest pain in the setting of a history of sickle cell disease with normal oxygen saturation and clear lung fields on chest x-ray. Osteomyelitis and painful vaso-occlusive crises represent the most common reasons for admission for sickle cell disease patients. Rib infarction may lead to a picture similar to ACS as the pain can lead to hypoventilation, which may result in atelectasis and the characteristic radiographic findings of ACS. However, no radiographic findings are present in this patient. As a result, rib infarction is the most likely etiology. C. Sepsis is incorrect because it would not typically present with isolated chest pain. Sepsis may often happen with sickle cell disease patients, given their susceptibility to infection. If cardiogenic shock follows, cardiomegaly may be appreciated. New infiltrates may occur if ARDS results secondarily from sepsis. However neither cardiomegaly nor new infiltrates were appreciated in this patient. While neither is required for the diagnosis of sepsis, the presence of fever may be the only indication of sepsis in a child with sickle cell disease. As a result, while less likely, sepsis is critical to include in the differential for this patient and to treat empirically until ruled out. D. CHF is incorrect because, like sepsis, it would not typically cause chest pain on its own. Additionally, abnormal lung markings and/or cardiomegaly would be expected in a patient with CHF. Neither is present in the patient in this vignette. E. Pneumothorax is incorrect because, other than rate, no abnormalities were noted on respiratory examination, and there were no supporting findings on chest x-ray. While a pneumothorax may cause chest pain, typically described as sharp and worsening with inspiration, this child's physical exam and radiographic findings make this option much less likely.

You are on the nursery service when your team is called to evaluate a 1-day-old infant. The infant was born via NSVD at 40 weeks' gestation to a 38-year-old G1P1A0 mother who did not have access to prenatal care and did not receive prenatal testing. The infant weighed 7 lbs 12 oz at birth and had Apgar scores of 7 and 8. On exam the infant is sleeping comfortably. She is afebrile with normal vital signs but appears to have low tone on exam. You also notice her ears seem to be lower than her eyes and appreciate mild edema of the hands and feet. Additionally, you note a fluid-filled sac at the base of the neck that does not appear to interfere with breathing. A karyotype performed after birth reveals a chromosomal abnormality. Which of the following is the most likely cause of this patient's condition? Single Choice Answer: Please select one answer. A Down syndrome B Turner syndrome C Fetal alcohol syndrome D Benign neonatal hypotonia E Cystic hygroma

B has been selected by the expert. A. Down syndrome is incorrect, because infants with Down syndrome typically have physical findings that include epicanthic folds, flat facial profile, single palmar creases, redundant neck skin, and heart defects. Other findings including a gap between the first and second toes. While infants with Down syndrome can be hypotonic as this infant is, the patient in this scenario does not have the other physical findings consistent with Down syndrome. B. Turner syndrome is correct. Turner syndrome is defined by the karyotype 45 XO. Characteristics of females with Turner syndrome include renal abnormalities, lymphedema (causing edema of hands and feet), low-set ears, congenital heart defects, dental abnormalities—such as narrow or high-arched palates—and cystic hygromas. Other possible physical findings include a webbed neck, widely-spaced nipples, and shield-like chest. As this mother did not have prenatal testing or a karyotype performed previous to her daughter's birth, one should be performed to confirm the diagnosis. C. Fetal alcohol syndrome (FAS) is incorrect. While FAS can cause hypotonia, this infant does not demonstrate the smooth philtrum, thin upper lip, or small palpebral fissures commonly seen in infants with FAS. FAS would not demonstrate a chromosomal abnormality on karyotyping. D. Benign neonatal hypotonia is incorrect because the infant demonstrates facial features that suggest an alternate etiology as well as abnormal karyotype. Typically, hypotonia of the newborn, especially at one day after birth, is abnormal and could reflect illness (such as sepsis), or neurologic dysfunction at either the central or peripheral levels. E. A cystic hygoma can present with a fluid-filled collection around the neck but is not associated with edema elsewhere and would not be associated with a chromosomal abnormality.

Devin is a 2-year-old African-American boy with sickle cell disease. Today his parents brought him to the ED because he was not feeling well. His mother reports that he has been very tired for the past week. Vitals show a temperature of 39°C, BP of 120/75 mmHg, RR of 24 bpm, and HR of 104 bpm. On physical examination he is ill appearing. His conjunctivae appear pale, and his sclerae are anicteric. Lungs are clear to auscultation. His abdominal exam is benign (non-tender, non-distended, with no organomegaly). His extremities are non-tender upon palpation. His nailbeds appear pale. Stat CBC reveals Hgb: 4.5 g/dL, Hct 15%, WBC 1800, and platelets 88,000. Mother is concerned as she has never seen him so ill before. What is the most likely cause of his new symptoms? A Vaso-occlusive crisis B Splenic sequestration C Aplastic anemia D Hemolytic crisis E Chest crisis

C has been selected by the expert. A. Vaso-occlusive crisis is a common acute concern in patients with sickle cell disease. This patient, however, does not complain of any pain. Vaso-occlusive crisis would likely present with pain, and is not usually associated with a decrease in all cell lines as seen in this child's CBC. B. Splenic sequestration could potentially cause thrombocytopenia and anemia in a sickle-cell patient; however, there is no organomegaly noted on exam, and it does not explain his fever. C. Parvovirus B19 causes aplastic anemia, especially in children with shortened RBC half-life. A mild anemia associated with parvovirus often goes unnoticed in healthy children. However, in patients with decreased RBC survival, such as sickle-cell patients, anemia can develop very rapidly and severely, along with decreases in the other cell lines (resulting in leukopenia and thrombocytopenia). In the above presentation, the presence of fever in a child with sickle cell disease should also raise concern for significant bacterial infection from encapsulated organisms like pneumococcus. This patient should have a blood culture and be started on empiric antimicrobial therapy pending screen labs and cultures. D. Hemolytic crisis is unlikely as the patient's sclera are anicteric, and thus he is probably not hemolyzing significantly. Hemolysis would also not explain his leukopenia or thrombocytopenia. E. Chest crisis should be considered in all sickle cell patients who present with chest pain, increased work of breathing, or infiltrates on chest x-ray, which may be consistent with either infection or infarction. The patient had no pulmonary symptoms, and the CBC findings are not consistent with chest crisis.

A 9-year-old female is brought to clinic by her mother because of two days of abdominal pain and vomiting. She has vomited six times today and has had decreased appetite, but no diarrhea, fevers, sick contacts, or changes in diet. Her mom states that she has been otherwise healthy apart from increased thirst and occasional bedwetting over the last few weeks. Of note, patient's maternal grandmother suffers from celiac disease. On exam, patient is afebrile and has a HR of 180 bpm, BP 90/60 mmHg, RR 50 bpm, and O2 saturation of 98%. She is lying in bed, appearing slightly drowsy, taking rapid, deep breaths and is slow to respond to questions. Her heart and lung exams are normal apart from being tachycardic, and abdominal exam reveals mild diffuse tenderness to palpation with no rebound or guarding. Which of the following would be the most appropriate next step in management? A Chest x-ray B Urine culture C Fingerstick glucose D Abdominal ultrasound E Gastric lavage

C has been selected by the expert. C. Obtaining a fingerstick glucose is the diagnostic step with the highest yield since the patient's clinical picture is strongly indicative of diabetic ketoacidosis (DKA). DKA is a condition more closely associated with Type 1 (rather than Type 2) diabetes, and is formally diagnosed if a random glucose is > 200 mg/dL, venous pH is < 7.3, bicarbonate is < 15 mEq/L and there is ketonemia or ketonuria. Patients in DKA can present with abdominal pain and vomiting secondary to metabolic acidosis that stems from ketonemia and lactic acidosis. Furthermore, osmotic diuresis from hyperglycemia may contribute to dehydration, which can manifest as tachycardia, hypotension and altered mental status. In an attempt to compensate for the metabolic acidosis, the patient may also present tachypneic with characteristic Kussmaul respirations (rapid, deep breaths). This patient's history of polydipsia, enuresis and family history of autoimmune disease (including celiac disease and Hashimoto's thyroiditis) suggest that the patient has Type 1 diabetes. Her current vital signs and general state of lethargy also point towards DKA and should be confirmed with a fingerstick glucose (in addition to other tests). A. A chest x-ray would be appropriate if bacterial pneumonia were high on your differential. A patient with pneumonia generally presents with fever, cough, tachypnea, and will likely have characteristic lung findings such as crackles on exam. The patient may experience abdominal pain secondary to pleural inflammation; however, vomiting is not a common presentation. The absence of fever, cough, sick contacts, and lung findings on exam make pneumonia a less likely diagnosis. B. Urine culture would be appropriate if pyelonephritis were high on your differential. A patient with pyelonephritis may present with history of fever, dysuria, urinary frequency, CVA tenderness, and vomiting. However, this patient's overall clinical picture does not support the diagnosis, since the patient is afebrile without a history of dysuria or classic CVA tenderness. D. Abdominal ultrasound would be indicated if appendicitis were high on your differential. Appendicitis may present with fever, nausea, vomiting, diarrhea, decreased appetite, and abdominal pain that localizes to the RLQ. Physical exam findings may include a positive Rovsing's sign or a positive psoas or obturator sign. Ultrasound may be a useful tool in detecting appendicitis in children. Appendicitis is less likely in this patient since she is afebrile and does not have the classic localizing pain characteristic of this condition. E. Gastric lavage would be the indicated next step of management if toxic ingestion is the confirmed diagnosis. A patient who has ingested toxic substance is usually afebrile and obtunded and may present with vomiting and manifestations of dehydration (secondary to vomiting). In a patient with salicylate toxicity, tachypnea is a common presentation. A thorough history to assess exposure to toxins and sending urine for evaluation of toxins may be appropriate to make this diagnosis. Given this patient's overall picture, DKA is a more likely diagnosis.

A 6-month-old infant arrives in the ED with a 12-hour history of poor feeding, emesis, and irritability. On exam, she is ill-appearing with T 39.2 C, P 160 bpm, R 40 bpm, BP 80/50 mmHg. CBC shows WBC 11.2, Hgb 13.5, Plt 250. Urinalysis shows > 100 WBC per hpf, positive leukocyte esterase, and positive nitrites. She has no history of prior urinary tract infection. Chest x-ray is negative. Urine and blood cultures are pending. After bringing her fever down, she was still uninterested in drinking, but her exam improved, and you were confident she did not have meningitis, so an LP was not performed. Which of the following is the best next step in management? Please select one answer. A Oral ampicillin B Oral ampicillin + gentamicin C Intravenous ciprofloxacin D Intravenous ceftriaxone E Intravenous piperacillin + tazobactam

D has been selected by the expert. D. This patient's presentation is suggestive of a UTI. Given the ill appearance, vital signs, and white count, Upper tract disease (pyelonephritis) should be strongly considered. A parenteral (IV/IM) third-generation cephalosporin is the best choice of those listed for pyelonephritis, given its excellent gram negative coverage (except for Pseudomonas). A. This patient is too sick for oral treatment, so oral ampicillin would be insufficient. Also, there is rising resistance of E. Coli to ampicillin, so secondary coverage with gentamicin or some other aminoglycoside would be needed unless cultures proved the organism was sensitive to ampicillin alone. B. Although parenteral and oral treatment produce similar outcomes in high quality RCTs, this patient is ill and refuses to drink and so requires parenteral antibiotics. IV ampicillin and gentamicin could be an appropriate choice for empiric coverage. C. Ciprofloxacin could be used for complicated UTIs, but it has the potential for adverse reactions in young children so is reserved for patients > 1 year with complications such as resistant organisms or urinary tract anomalies E. IV piperacillin + tazobactam has excellent gram negative coverage with added Pseudomonas coverage, but it is highly expensive and Pseudomonas is unlikely to be the cause of a UTI in a child who is not regularly catheterized.

When you are at your preceptor's office, you are told to see Amy, a 20-month-old female diagnosed with cerebral palsy (CP) at the last visit. Your preceptor knows that you just had your lecture on CP and tries to have you figure out which type of CP Amy has. She tells you that Amy was born at full term but had severe jaundice and required extensive treatment for hyperbilirubinemia, including an exchange transfusion. She now has slow and uncontrolled movements throughout her body. Her brain MRI shows some atrophy of the basal ganglia. What type of CP does Amy have? Single Choice Answer: Please select one answer. A Dyskinetic cerebral palsy B Spastic diplegia C Spastic quadriplegia D Spastic hemiplegia E Ataxic cerebral palsy

A has been selected by the expert. A. This choice is correct because dyskinetic CP is associated with kernicterus, due to hyperbilirubinemia, as well as findings of basal ganglia pathology on imaging. Patients typically have motor abnormalities throughout the body. Dyskinetic CP is also associated with perinatal asphyxia and can involve the thalamus and cerebellum on imaging. B. This choice is incorrect because spastic diplegia is classically associated with premature birth and specific MRI findings of periventricular white matter abnormalities. Patients present with motor involvement that is more prominent in the legs than the arms. C. This choice is incorrect because patients with spastic quadiplegia have spasticity, clonus, and exaggerated tendon jerks throughout their bodies. Imaging would show global brain abnormalities. D. This choice is incorrect because spastic hemiplegia—associated with a stroke damaging a unilateral upper motor neuron tract—should present with spasticity of the contralateral arm and leg, not the entire body. E. This choice is incorrect because ataxic cerebral palsy should show cerebellar abnormalities on imaging, while Amy's MRI shows only basal ganglia involvement.

A 4-week-old female infant presents to clinic for a well child check. This infant had an uneventful delivery by NSVD at full term and subsequent normal neonatal screen. The nurse reports that her growth is a concern, with weight at 3.0 kg (< 3rd percentile) and weight for height at < 3rd percentile. Mom denies any drinking or drugs since before this pregnancy and says she has been breastfeeding every two to three hours and supplementing with appropriately mixed formula one to two times a day. She does report the baby seems to have issues latching and some possible gasping between suckles. There has been no diarrhea, hematochezia, vomiting, or fevers. The vital signs and exam (apart from a thin infant) are normal. The mother's affect is flat, and she seems anxious when you ask her about her infant. What is the most likely diagnosis for this infant's failure to thrive? Single Choice Answer: Please select one answer. A Malabsorption B Gastroenteritis C Milk protein allergy D Congestive heart failure E Failure to thrive due to inadequate caloric intake

E has been selected by the expert. A. Malabsorption typically presents with poor weight gain and loose stools despite good caloric intake. This infant has poor caloric intake with constipation, making this diagnosis unlikely. B. Gastroenteritis typically presents with loose stools, emesis, and fever. This infant has none of these symptoms. C. Milk protein allergy typically causes bloody stools rather than isolated poor weight gain. The patient does not have bloody stools or other signs of allergy or formula intolerance. Milk protein allergy is less common but still occurs in breastfed infants. D. Congestive heart failure is important to consider in any child with failure to thrive, but this child has no signs of CHF such as tiring with feeds, sweating, or tachypnea, tachycardia, or hepatomegaly. E. This infant is likely not consuming adequate calories to grow. The mother or primary caregiver may neglect proper feeding of the infant because of preoccupation with the demands or care of others, her own emotional problems, substance abuse, lack of knowledge about proper feeding, or lack of understanding of the infant's needs. It is also important to assess the mother for post-partum depression. Standardized screening tools, such as the Edinburgh, are used routinely in pediatric office settings for this purpose.

A 56-year-old male presents for care at the ED complaining of dry cough for the past three days. He notes that this problem started a few days after his family's annual fish fry and barbecue and has been worsening since. He has no known past medical history but mentions that he has not seen a doctor in years. He notes that the cough is worse at night often waking him from sleep. He is unable to lie flat on his back and has started using three to four pillows to sleep comfortably. He also reports increased swelling in his legs that worsens throughout the day. He denies having any chest pain or palpitations and also does not believe he has had any sick contacts. He does not know his family history since he was adopted as a child. He has not had any fevers, sweats, or chills. On exam, you observe a tachypneic, obese man in mild distress. On chest auscultation, he has an S3, bilateral rales at the lung bases, and 2+ pitting edema in the lower legs bilaterally. What is the most likely diagnosis? A New onset heart failure B Pneumonia C Sleep apnea D Anxiety

The correct answer is A. The patient's description of the cough (worse at night, unable to lie flat, wakes from sleep), physical exam findings (obesity, rales in the lung bases, pitting edema), and history (started after a fish fry/BBQ) point to CHF as the most likely diagnosis. A lack of fever points away from pneumonia and a lack of description of snoring or pauses during sleep points away from sleep apnea. Anxiety would not cause the symptoms or physical exam findings described here.

A 56-year-old male presents for care at the ED complaining of dry cough for the past three days. He notes that this problem started a few days after his family's annual fish fry and barbecue and has been worsening since. He has no known past medical history but mentions that he has not seen a doctor in years. He notes that the cough is worse at night often waking him from sleep. He is unable to lie flat on his back and has started using three to four pillows to sleep comfortably. He also reports increased swelling in his legs that worsens throughout the day. He denies having any chest pain or palpitations and also does not believe he has had any sick contacts. He does not know his family history since he was adopted as a child. He has not had any fevers, sweats, or chills. On exam, you observe a tachypneic, obese man in mild distress. On chest auscultation, he has an S3, bilateral rales at the lung bases, and 2+ pitting edema in the lower legs bilaterally. What diagnostic test would you perform first? A Exercise stress test B Pharmacologic stress test C Echocardiogram D EKG

The correct answer is D, an EKG can quickly help determine whether the patient is in sinus rhythm, whether there is ischemia or infarction, or whether there is Left Ventricular Hypertrophy. These findings may help you determine the etiology of this patient's CHF. Other tests (especially an echocardiogram) may be indicated at some point during the patient's work-up, but may not be appropriate first tests to order. Specifically, cardiac stress testing is more useful in ascertaining the diagnosis and prognosis of coronary artery disease than in congestive heart failure, although they are often a concurrent part of an extensive workup.

Mr. Jones is a 63-year-old male with no significant past medical history who presents with increased swelling in his legs and shortness of breath. He reports that he has smoked two packs per day for the past 20 years. He does not exercise regularly and notes that he has always been overweight. Mr. Jones notes that his maternal uncle died of a heart attack at age 43. He denies having any chest pain or palpitations and his exam is notable only for 2+ pitting edema in the lower extremities. His most recent labs show HDL of 50, LDL 101, and a blood glucose of 112. Which of the following risk factors has most likely contributed to Mr. Jones' problem? A Diabetes B Elevated HDL C Heart attack in uncle D Male gender

The correct answer is D, male gender is Mr Jones' main risk factors for CAD. With a blood glucose of 112, he is in the pre-diabetic range, but does not have a formal diagnosis of diabetes. His HDL is on the high side, but this is a protective factor. His uncle, though he did have a heart attack at a young age, is not a first-degree relative.

A 2-year-old male presents to the ED with a 5-hour history of hyperactivity, fever, and sweating. His BP is 160/90 mmHg, HR 130 bpm, RR 30 bpm. On exam, he has dilated pupils, cool skin, and hyperreflexia. What is his most likely accidental medication ingestion? A Pseudoephedrine B Codeine C Iron pill D Acetaminophen E Propranolol

A. Choice A is correct. Ingestion of a sympathomimetic like pseudoephedrine stimulates the beta and alpha adrenergic receptors, causing elevated HR, RR, BP and hypothermia along with diaphoresis, dilated pupils, hyperreflexia, and hyperactivity. B. Choice B is incorrect because constricted pupils are seen in ingestions of opioids. Slowed breathing, rather than tachypnea, may also be seen. C. Choice C is incorrect. Iron toxicity would present with severe abdominal symptoms followed by signs of shock. D. Choice D is incorrect. Acetaminophen toxicity initially presents with minimal symptoms, followed by symptoms of liver toxicity such as jaundice and RUQ pain. E. Choice E is incorrect. Beta-blocker toxicity would cause bradycardia, not tachycardia and hypertension.

A previously healthy and developmentally normal 16-month-old male comes to the urgent care clinic with his father with a chief complaint of his first reported seizure. The child was reported to have dropped to the floor with loss of consciousness and had sporadic twitchy movements of his legs and arms that lasted for five minutes. The child has had URI symptoms for the past two days, with a fever to 103 degrees F without any changes in mental status. Neither parent has a seizure disorder, but the child's mother reports having a single seizure as a young girl once after developing a high fever after a cold. What is the most likely diagnosis? A Epilepsy B Simple febrile seizure C Complex febrile seizure D Absence seizure E Cyanotic breath-holding spell

B has been selected by the expert. A. Choice A is incorrect because epilepsy is a recurring condition, while this seizure is this infant's first episode. In addition, the infant has had URI symptoms with a high fever, likely leading to a febrile seizure. Epilepsy seizures are usually classified as unprovoked. If this is indeed a simple febrile seizure, the child's risk of developing epilepsy in the future is about 0.5-1% above the normal risk. If this child continues to have febrile seizures or starts experiencing complex febrile seizures, the risk for epilepsy in the future is even greater. B. Choice B is correct. Febrile seizures are one of the most common causes of seizures in children. Simple febrile seizures are more common than complex febrile seizures and are characterized by < 15 minutes duration, occurring only once in a 24-hour period, and are generalized (in this patient's case, generalized tonic-clonic). Febrile seizures are usually self-limited events triggered by an acute febrile illness. A positive family history for febrile seizures in the parents makes it a more likely diagnosis in their children. This particular diagnosis fits this patient the best. C. Choice C is incorrect because a complex febrile seizure is characterized by > 15 minute duration, occurring more than once during a 24-hour period, and being focal. Given that this child's seizure was associated with loss of consciousness and tonic-clonic movements lasting five minutes, and occurred only once during a 24-hour period, complex febrile seizure is not the most likely diagnosis. D. Choice D is incorrect because absence seizures are generalized seizures, but children recover consciousness much more quickly, usually within 30 seconds, compared to tonic-clonic seizures, which take much longer. Absence seizures are most often seen in children starting from the age of 3 and are not associated with loss of tone. Potential causes include hyperventilation or photic stimulation. Given that this child's seizure occurred after URI symptoms and high fever and also was characterized by tonic-clonic jerks lasting about five minutes, this is not the most likely diagnosis. E. Choice E is incorrect. Breath-holding spells are most commonly seen in infants from 6 months to 6 years of age. The pathogenesis is not entirely understood but may involve autonomic nervous system dysfunction. In a cyanotic breath-holding spell the infant becomes angry or upset and there is a period of crying, followed by breath-holding in a forced expiration state that may lead the child to lose consciousness and become cyanotic. The child usually recovers soon thereafter with no residual side effects. Given that this child was not reported to have been angry or crying prior to losing consciousness and did not become cyanotic, and because he demonstrated tonic-clonic jerking movements for five minutes, a breath-holding spell would not be a likely diagnosis.

A 12-year-old male presents to the ED with complaints of anorexia, weight loss, and persistent cough, with nocturnal coughing fits that have been waking him from sleep for the past three weeks. He denies fever, chills, myalgia, sore throat, or rhinorrhea. The patient presented to his primary care physician one week prior with the same complaint, and was treated with amoxicillin and bronchodilator therapy. His chest x-ray was negative for infiltrates at that visit. The patient's symptoms did not improve with this regimen. The cough became more frequent, sometimes causing emesis. Which of the following is the most likely diagnosis? A Reactive airway disease B Infection with Bordetella pertussis in the catarrhal stage C Infection with Bordetella pertussis in the paroxysmal stage D Atypical pneumonia due to Mycoplasma pneumoniae E Laryngotracheobronchitis

C. The paroxysmal stage of pertussis lasts four to six weeks and is characterized by repetitive, forceful coughing episodes, followed by massive inspiratory effort. This massive inspiratory effort is what results in the characteristic "whoop"-sounding cough. This is consistent with the patient's presentation and duration of illness. The forceful coughing fits in pertussis can even lead to conjunctival hemorrhages and pneumothoraces from the increased intrathoracic and intracranial pressures from Valsalva. The antimicrobial agents of choice for treatment of pertussis are azithromycin, clarithromycin, and erythromycin. Antibiotics given in the paroxysmal phase will reduce communicability but will not alter the clinical course. A. Reactive airway disease would most likely have improved with bronchodilator therapy. In addition, you would not expect to see anorexia or weight loss with reactive airway disease. B. The catarrhal stage of pertussis lasts one to two weeks and is often indistinguishable from URI. In this patient the presenting symptoms have been an ongoing problem for more than three weeks. D. Mycoplasma pneumonia would be expected to be associated with fevers and findings on chest x-ray and lung exam. E. Croup (laryngotracheobronchitis) presents with difficulty breathing and a "seal bark" cough and usually lasts a week or less. Croup also is usually associated with fever.

A 4-year-old boy who recently emigrated from eastern Europe presents with his mother to your general pediatrics clinic. His mother reports that he has a chronic nonproductive cough during the day and night, mild wheezing for one month and failure to gain weight (his weight has dropped from the 50th to the 10th percentile for his age). His mother denies any high fevers, rhinorrhea, or night sweats. Which of the following are the next best diagnostic tests? A Chest x-ray and tuberculin skin test B CT of nasal sinuses C Spirometry, before and after bronchodilator therapy D Chest x-ray and methacholine challenge E None needed, patient likely has habitual cough

A has been selected by the expert. A. CXR and tuberculin skin test (TST) is the best choice. Signs and symptoms of primary pulmonary tuberculosis are few to none. Toddlers may present with nonproductive cough, mild dyspnea, wheezing, and/or failure to thrive (defined as weight < 5th percentile or drop in two percentile curves for weight). In children, TB can present without systemic complaints (fever, night sweats, and anorexia), severe cough, and sputum production. Regarding diagnostic tests, the TST is a practical tool for diagnosing TB infections. All children with chronic cough (more than three weeks) should be evaluated with a chest x-ray, as other pathology—such as lung abscess or malignancy—can also be detected on CXR. B. Sinusitis is often preceded by a URI, with nasal congestion as a prominent feature, leading to nocturnal cough due to post-nasal drip. These symptoms are not seen in our patient. Furthermore, a diagnosis of sinusitis is made clinically, with CT scan obtained only in complicated cases or cases resistant to treatment. Complications include cavernous sinus thrombosis, meningitis, and epidural abscess. C. Spirometry (pulmonary function testing) before and after bronchodilator therapy is the most specific means of determining whether or not a child has reactive airways. Asthma is a very common diagnosis in pediatrics, and may present with cough that is worse at night and exacerbated by exercise and cold air. Patients with cough-variant asthma present with only cough, typically nonproductive. However, given this patient's failure to thrive, a more serious diagnosis such as TB must be considered. Also, a chest x-ray is needed in all children with chronic cough (more than three weeks). D. Although a chest x-ray is appropriate in all children with chronic cough, a methacholine challenge (for asthma) would be inappropriate in this scenario. Although asthma is a common diagnosis, given the patient's failure to thrive, a more serious diagnosis must be considered. Further, a methacholine challenge is reserved for cases in which asthma is suspected and spirometry is normal or near normal, and should be performed by trained individuals. E. Habitual cough is caused by habitual perpetuation of a cough that begins with a viral URI. Continued coughing further irritates the airway, leading to stronger stimulation to cough. The cough is typically very loud, short, dry, brassy, and spasmodic. This cough is unchanged by exercise or cold air, and classically resolves during sleep. Although the patient in this case has a dry cough, his failure to thrive points to a more serious diagnosis (e.g., TB). All children with chronic cough (persisting longer than three weeks) need a CXR.

A 2-year-old male is brought into the ED by his mother because of vomiting and altered mental status. He has pinpoint pupils and seems to be drooling and sweating uncontrollably. His heart rate is 60 bpm, his respiratory rate is 45 bpm, and he seems to have difficulty breathing. Which ingestion is the most likely cause of his symptoms? A Organophosphates B Tricyclic antidepressant C Barbiturates D Codeine E Pseudoephedrine

A. Organophosphates cause cholinergic effects, such as miosis, sweating, lacrimation, salivation, urination, increased gastric mobility (vomiting, diarrhea), muscle twitching, bronchospasm, bradycardia, and seizures. A good mnemonic is SLUDGE (salivation, lacrimation, urination, defecation, GI mobility, emesis). B. Tricyclic antidepressants are part of the group of anticholinergics. Overdose can cause mydriasis, dry and red skin, fever, delirium, seizures, tachycardia, urinary retention, and ileus. C. Barbiturates are part of the group of sedative-hypnotics. Excessive ingestion presents as blurred vision, hypotension, apnea, bradycardia, hypothermia, sedation, delirium, and coma. D. Codeine and other opioids can cause miosis, respiratory depression, bradycardia, hypotension, hypothermia, and depressed mental status. E. Pseudoephedrine is a sympathomimetic, and overdose results in mydriasis, fever, diaphoresis, tachycardia, agitation, and seizures

You are notified that a 10-day-old patient in your practice had a newborn hemoglobin screen positive for sickle cell disease. Pregnancy and delivery were uncomplicated. Mother is 19 years old and works as a nurses' assistant at a nursing home. When questioned, she says she remembers her grandmother died of chest pain and a lung infection. Which of the following should be ordered next for the baby? A Antibiotics B CBC C Chest x-ray D H. influenza type b vaccination E Transcranial Doppler

A has been selected by the expert. A. Correct. This newborn has sickle cell disease. Her grandmother likely passed away from acute chest syndrome, a common complication of this disease. Due to decreased splenic function and consequent decreased resistance to infection with encapsulated organisms (Streptococcus pneumoniae, Haemophilus influenzae type b, Neisseria meningitidis), infants and young children with sickle cell disease are at increased risk for sepsis. When given to infants with sickling disorders, penicillin significantly decreases the risk of mortality from overwhelming sepsis. Prophylaxis is usually continued until the child is five or six years of age (after this, there is little data to support its use except in patients who have had documented sepsis and bacteremia, or who have had their spleens removed). B. A CBC is not warranted immediately in the absence of symptoms of infection in a newborn. C. A chest x-ray is not indicated in this infant with no respiratory symptoms. D. Vaccination against Haemophilus influenzae and S pneumoniae (with the 13-valent conjugate vaccine Prevnar 13) is indicated at 2, 4, and 6 months of age. The meningococcal vaccine is indicated at 2 years of age for patients with functional asplenia, with a booster at 5-6 years of age. The 23-valent pneumococcal polysaccharide vaccine is indicated at 2 years and between 5 and 8 years of age. This infant is too young for administration of the Hib vaccine. E. A transcranial Doppler may be performed in patients with sickle cell disease to assess for stroke risk but is not indicated in an asymptomatic newborn.

You see a 6-year-old male in the ED who presents with a history of a 10-second episode of jerking movements of his extremities with unresponsiveness, observed by both of his parents. His parents claim he has had abdominal pain and small quantities of bloody diarrhea for two days. The child has no significant past medical history, has taken no medications recently, has no pets, and has not traveled outside of California in the past year. He attends kindergarten. Which organism is the most likely cause of the child's symptoms? A Shigella sonnei B Rotavirus C Clostridium difficile D Enterotoxigenic E. coli (ETEC) E Vibrio cholerae

A has been selected by the expert. A. Shigella sonnei causes bloody diarrhea and WBCs in the stool on Wright stain. Rarely, children infected with Shigella can suffer from seizures due to neurotoxin release. B. Rotavirus is the most common cause of infectious gastroenteritis. It is an RNA virus that does not cause WBCs to appear in the stool, and typically does not cause bloody diarrhea. Diagnosis is made using ELISA. Rotavirus is not associated with seizures. C. Clostridium difficile typically causes diarrhea after antibiotic use, which allows the organism to overgrow the normal intestinal flora. It would not be expected to cause diarrhea in an otherwise healthy child who is not taking any medications. Infection can be diagnosed by the presence of C. difficile toxin in the stool. D. ETEC is the strain of E. coli that commonly causes traveler's diarrhea, a noninvasive infection without WBCs in the stool. Diarrhea is typically watery and contains no blood. E. Vibrio cholerae causes massive quantities of watery diarrhea, and patients may present with severe dehydration. Bloody diarrhea would not be expected. It is typically seen in endemic or epidemic form in developing countries, and would not be expected in a child living in the U.S. with no recent travel history.

A 7-year-old boy presents with a five-year history of intermittent vomiting, vertigo, and throbbing unilateral headaches that seem to be induced by emotional stress and when his teacher wears perfume. He reports that the pain is not worsened by long naps or coughing. His mother reports that she has a history of headaches that started as a child and wonders if her son inherited this from her. His neurological exam shows no focal deficits. What is the next step in diagnosis or treatment? Single Choice Answer: Please select one answer. A Trial of prophylactic medication for migraine headaches B MRI C NSAIDs D Referral to an ENT surgeon

A has been selected by the expert. A. This child is presenting with signs of both typical and atypical migraines and could be started on a trial of prophylactic medication. Tricyclic antidepressants (TCAs) are often used in children for migraine prophylaxis, which is the most likely diagnosis in a child with this constellation of symptoms. B. An MRI would be indicated if an intracranial mass is suspected. An infratentorial mass, which could increase intracranial pressure, might cause headaches in the morning after laying down, or also when valsalva'ing during a cough. An infratentorial mass can also compress the cerebellum and may cause ataxia, dysarthria, and nystagmus. A supratentorial mass is more likely to present with focal motor and sensory deficit. C. NSAIDs are often used for tension headaches. Unlike migraines, however, tension headaches cause pain in a band-like pattern around the head. D. While ENT doctors specialize in vertigo that is caused by inner ear pathology, it is likely that this patient's vertigo is due to an atypical migraine.

An 18-month-old male comes to the clinic for a well-child check. His mother says he is a happy boy and endorses no complaints. She notes that he was born a few weeks early via emergency C-section and spent two months in the NICU. She says he is feeding well and gaining weight. When you ask about developmental milestones, you find out that that he pulls himself up to stand, but is not walking yet. He has several words and is interactive. There is no family history of disease during infancy. He is up to date on his immunizations. He is one of five children and his parents are currently experiencing difficult financial times. Exam reveals increased tone and hyperreflexia in his lower extremities. What is the most likely diagnosis of his developmental delay (if any)? Single Choice Answer: Please select one answer. A Cerebral palsy B Normal variant C Metabolic disorder D Genetic abnormality E Psychosocial stress reaction

A has been selected by the expert. A. This choice is correct. Children with cerebral palsy often present with defects in motor development and are often found to have abnormal neuromuscular exams, including increased tone and reflexes. Other areas of development are less consistently impacted. The cerebral palsy most likely was the result of an hypoxic injury during the perinatal period. B. This choice is incorrect because this child has profound motor delay and abnormal neurologic findings (hypertonia). C. This choice is incorrect because this child's newborn screening exam was normal, he has no family history of metabolic disease, and has not had any indication of any metabolic abnormalities. D. This choice is incorrect because of a lack any characteristic syndromic phenotype and negative family history, and the delay appears to be limited to his gross motor exam. E. This choice is incorrect because of his persistence of developmental delay and lack of behavioral changes (temper tantrum, sleep disturbance, etc.). Temporary loss of developmental achievements can be seen in times of stress.

Why is it important to distinguish between cardioembolic etiologies of stroke versus atherothrombotic? A Cardioembolic etiologies require treatment with oral anticoagulant therapy B Distinguishing between these two etiologies is not particularly important because treatment for either is the same; anti-platelet C Distinguishing between these two etiologies is not particularly important because treatment for either is the same; anticoagulant D Atherothrombotic etiologies require treatment with oral anticoagulant therapy

A has been selected by the expert. The correct answer is A. Finding the cause of a stroke is important because the source of the stroke can help determine the treatment. For patients with a cardioembolic cause, they should be placed on coumadin therapy. For patients with an atherothrombotic stroke, they need to be placed on an anti-platelet agent. Both of these treatment options are intended to reduce the risk of stroke

Which of the following therapies are recommended as first-line therapy for secondary prevention of noncardioembolic TIA or noncardioembolic stroke? A Aspirin B Coumadin C Atorvastatin D Recombinant tissue plasminogen activator (t-PA)

A has been selected by the expert. The correct answer is A. For noncardioembolic (or atherthrombotic, lacunar) strokes, possible first-line therapies for secondary prevention include aspirin, aspirin in combination with extended-release dipyridamole (aggrenox), and plavix. For cardioembolic strokes, aspirin may potentially be desirable in patients with a contraindication for anticoagulation, but most people will need to be on coumadin for secondary prevention of stroke. Statins are used to help treat high cholesterol in patients who have had a stroke or have risk factors for stroke. T-PA is used in the acute management of a stroke and not as secondary prevention of noncardioembolic stoke.

A 68-year-old male becomes confused and agitated on post-operative day three after open cholecystectomy. He does not recall his surgery or where he is and wants to leave the hospital. Physical exam shows T=99.9, BP=143/89, HR=90, RR=13, O2 sat: 98% on room air. He appears agitated and uncomfortable. He has a Foley catheter. His physical exam is unremarkable. Labs: WBC-11,000, Urinalysis: 2+ nitrites and 2+ leukocyte esterase, 10 WBC/hpf. Glucose finger stick: 80, EKG: normal sinus rhythm at 88 beats per minute. Which of the following is the best initial step in the management of this patient? Single Choice Answer: Please select one answer. A Remove urine catheter B Give IV ceftriaxone C Order urine gram stain D Order urine culture E Give oral ceftriaxone

A has been selected by the expert. The correct answer is A. This patient has an UTI which is the most common cause of delirium in the elderly. Catheters increase the incidence of UTIs and removal would be the appropriate first step. The other options would be appropriate to consider once the catheter is removed.

Which of the following statement is true regarding TIAs when compared to infarction? A A brief episode, lasting < 24 hour, of neurologic dysfunction caused by focal brain ischemia with no evidence of acute brain infarction B A brief episode, lasting <48 hours, of neurologic dysfunction caused by focal brain ischemia with no evidence of brain infarction C A neurologic episode lasting >24 hours of presumed vascular origin leading to a sudden onset of neurologic impairment D A sudden onset of neurologic impairment lasting > 48 hours associated with sudden onset of facial weakness

A has been selected by the expert. The correct answer is A. Transient ischemic attacks are defined as episodes of neurologic dysfunction caused by focal brain or retinal ischemia with clinical symptoms lasting less than 24 hours and no evidence of acute infarction on imaging. Stroke is defined as sudden onset of focal (or global) of neurologic impairment with symptoms lasting more than 24 hours.

A 16-year-old female presents with acute onset of diffuse abdominal pain with periodic sharpness in the right upper quadrant that radiates to her back. She has had some episodes of vomiting and has a fever. She is sexually active and has used alcohol in the past. Which of the following is most likely to present with right upper quadrant pain? A Pancreatitis B Urinary tract infection C Ectopic pregnancy D Appendicitis E Ovarian torsion

A has been selected by the expert. A. Choice A is correct because pancreatitis commonly causes continuous abdominal pain that can localize to the right and left upper quadrants ("band-like pain") as well as radiating to the back. Nausea and vomiting are nearly always present. Lipase is the most sensitive and specific lab test to diagnose pancreatitis. B. Choice B is incorrect because urinary tract infections classically cause painful urination with frequency and urgency. Patients may present with suprapubic tenderness, and flank pain is common when the infection has spread to the kidney. UTIs are common in sexually active females, and fever and vomiting are often seen with pyelonephritis, but sharp right upper quadrant pain would not be expected. C. Choice C is incorrect because although ectopic pregnancy is always a concern in a sexually active female presenting with abdominal pain, the pain is typically located in the lower abdomen. Fever and diffuse abdominal pain are uncommon in uncomplicated ectopic pregnancy. D. Choice D is incorrect because appendicitis classically starts as periumbilical pain that migrates to McBurney's point (1-2" from the anterior superior iliac spine toward the umbilicus). It is also of concern in a teenager with a presentation like this, as acute appendicitis is the most common pediatric condition requiring emergency surgery. Of children presenting with acute abdominal pain, 1-4% have appendicitis. E. Choice E is incorrect. Although ovarian torsion is possible at any age, the abdominal pain is typically described as stabbing and is usually localized to the lower abdomen and pelvis. Nausea and vomiting can also be present.

A 3-month-old male presents to the ED with a fever that started the previous day. Mother reports that he was fussy and had decreased oral intake. He had had five fewer diaper changes than usual. He had no vomiting, diarrhea, or respiratory difficulty. On physical exam his temperature is 101.6 F, pulse 110 bpm, RR 24 bpm, and BP 95/67 mmHg. The baby seems irritable and is not consolable by the parent. HEENT exam was significant for dry mucous membranes. Other than his irritability, the rest of the physical exam was unremarkable. CBC showed WBC 3.5, but was otherwise normal. BMP was within normal limits. Urinalysis showed positive leukocyte esterase, positive nitrite, and WBCs > 10/hpf. An LP was performed, and urine and CSF culture results are pending. The patient is placed on IV fluids and is started on cefotaxime. What is the next best step in evaluation? Please select one answer. A Renal bladder ultrasound B Kidney-ureter-bladder (KUB) x-ray C Intravenous pyelogram D VCUG E Oral ampicillin

A has been selected by the expert. A. This infant has a fever without other respiratory symptoms. Meningitis and UTI must be considered in patients with fever. The only way to rule out meningitis is by lumbar puncture. This patient has a low WBC, suspicious for sepsis, and a UA that is highly suggestive of UTI. Empiric therapy should be started to cover common organisms including E.coli, P. mirabilis, and Klebsiella. Cefotaxime is reasonable empiric therapy. Renal ultrasound is recommended for all infants with pyelonephritis to assess for renal structural abnormalities or signs of obstructive uropathy (hydronephrosis). B. KUB is not recommended for UTI. C. Intravenous pyelogram would expose the patient to radiation and would not be recommended to screen for renal abnormalities. D. VCUG screening is recommended only for recurrent UTI or when there is abnormal renal ultrasound. E. The patient is already on parenteral antibiotics, so oral antibiotics would not be necessary. Also, ampicillin would not provide empiric coverage.

You are working overnight call in the ED when Charlie, a 3-year-old male infant, arrives after his parents witnessed an episode of convulsions at home. His parents report that Charlie was in his usual state of good health until three days ago when he developed fever, cough, and rhinorrhea. This evening they found him in bed with his eyes rolled upward, jerking all four of his extremities uncontrollably. He was unarousable from this state, which self-resolved after about two minutes. This has never happened before. Currently, Charlie is sleepy but arousable and complains of nausea. His vitals include T 103.2 F, P 112 bpm, BP 100/60 mmHg, RR 22 bpm, O2 sat 99% on room air. Aside from rhinorrhea and erythematous mucous membranes, the remainder of his physical exam is unremarkable. What is the next best step in management? A Workup for source of fever B EEG C MRI brain D Abdominal ultrasound E Administration of valproic acid

A has been selected by the expert. A. This is the correct answer because Charlie has likely experienced a febrile seizure in the setting of an infection. These are relatively common occurrences in Charlie's age range (6 months to 5 years), and the first priority would be to identify the source of fever and treat it. B. This answer is incorrect because EEGs are indicated in recurrent, focal, or complex seizures. An EEG may eventually be useful to evaluate Charlie's situation, especially if he has another seizure in the ED or if his febrile seizures recur in the future; however, identification of the source of the fever should be the first priority. In addition, interictal EEGs are only positive in 60% of children with epilepsy. C. This answer is incorrect. In the differential diagnosis of a self-limited generalized seizure in the setting of fever in a child of this age group, a febrile seizure is higher on the differential than a brain mass. D. This answer is incorrect. Acute abdomen is low in the differential in the setting of the presenting symptoms of cough, rhinorrhea, and fever. E. This answer is incorrect, because the majority of patients with febrile seizures do not require treatment with anticonvulsants. This is especially true if this is Charlie's first febrile seizure. Anticonvulsants such as phenobarbital or valproic acid may rarely be indicated, but generally these drugs are not recommended in the setting of febrile seizures because they are associated with serious side effects.

You are working in the pediatric ED when a 3-year-old girl, Jenny, presents with altered mental status for the past six hours. Her mother reports that the babysitter called her at work today after Jenny started acting agitated and "looking very sick." The mother reports "she feels so warm, I think she has a fever and has become dehydrated." On exam, the patient is agitated and anxious with dilated pupils. Her skin is warm and dry. Vitals reveal tachycardia and hypotension. You suspect the child may have accidentally ingested one of her mother's medications. An overdose of which of the following medications could cause Jenny's symptoms? A Tricyclic antidepressant B SSRI C Decongestant D Acetaminophen E ACE inhibitor

A. Tricyclic antidepressant (TCA) is correct. TCA toxicity presents with agitation, tachycardia, hypotension, dilated pupils, and hot, dry skin from the anticholinergic effects of TCAs. B. SSRI is incorrect, because serotonin syndrome will usually present with a fever and sweaty, not dry, skin. Also, serotonin syndrome usually does not present with hypotension. C. Decongestant is not the correct answer. Decongestant overdose will elicit a sympathomimetic response, including agitation and tachycardia. However, unlike TCA overdose, decongestant overdose will cause hypertension, constricted pupils, fever/sweating. Additionally, severe overdose of decongestant may cause seizures. D. Acetaminophen is not the correct answer. Acetominophen overdose will produce mild and nonspecific symptoms and include right upper quadrant pain, with elevation in liver enzymes, resulting in liver failure at toxic levels. E. ACE inhibitor is not the correct answer. The typical adverse event for ACE inhibitors is cough.

A 6-month-old baby boy is referred to your clinic because he has not been gaining weight appropriately. His mother denies any difficulty with feeding or reduced appetite, yet his weight has still dropped from the 30th to the 3rd percentile. Mother also complains that he has loose, malodorous stools. After a thorough workup, a diagnosis of cystic fibrosis (CF) is made. Which of the following is a TRUE statement regarding CF? Single Choice Answer: Please select one answer. A CF is an autosomal dominant disorder B CF is caused by a mutation in CFTR, resulting in defective salt balance C CF is a disease that exclusively involves the respiratory system D Gene therapy is now the primary source of CF therapy E It is important to provide calories at a lower level than recommended dietary allowance for a given age in order to prevent GI upset

B has been selected by the expert. A. CF is autosomal recessive, NOT dominant. Most people with CF do not have a positive family history. B. Correct. The mutation in CFTR gene results in defective salt balance. CF is caused by mutations in a single large gene on chromosome 7 that encodes the cystic fibrosis transmembrane conductance regulator (CFTR) protein. Clinical disease requires disease-causing mutations in both copies of the CFTR gene. C. CF does involve the respiratory tract—leading to chronic cough and recurrent infections—but may also lead to pancreatic dysfunction, infertility, malnutrition, and more. D. Gene therapy is NOT a current treatment option for CF patients. However, there are studies right now that are investigating gene therapy. Current treatment includes a multidisciplinary approach, using treatments such as pancreatic enzyme and vitamin replacement, mucus clearing, possibly transplant, and more. E. It is necessary to provide calories at a HIGHER level than the recommended dietary allowance for age to maintain appropriate nutritional status, as CF patients usually have a malabsorption problem.

The mother of a 5-year-old boy calls your office asking if she should take her son to the emergency room or wait another day. She states that her son suddenly developed a "high fever" and is extremely tired. When you ask about her son's behavior, she states that he also seems very confused. She also noticed he had developed reddish-purplish spots on his extremities. What is the next best step in management of this patient? Single Choice Answer: Please select one answer. A Have the patient make an appointment to come to your office today B Tell the mother to take her son to an ED immediately C Have the patient hydrate well over the weekend and follow up with you in a few days D This patient most likely ingested something. Recommend ipecac to induce emesis and call 911

B has been selected by the expert. A. Choice A is incorrect because any acutely altered mental status should raise serious concern, and therefore an office visit would probably not be adequate. B. Choice B is correct. This patient is exhibiting signs of sepsis, more specifically, of meningococcemia. Although it is important to replenish this patient's fluids and control his fever, it should not be done in an outpatient setting. This is a medical emergency! Sepsis can lead to altered mental status. Signs and symptoms of sepsis include: fever, nausea, vomiting, diarrhea, apnea/dyspnea, oliguria, pallor, tachypnea, tachycardia, lethargy, irritability, petechiae, purpura, tremors, and seizures. C. Choice C is incorrect. This patient is too ill to remain at home over the weekend with just supportive care. If he is bacteremic and septic he has a high mortality rate over the next 12 hours. D. Choice D is incorrect. There is no indication for inducing emesis, and this can present a risk of aspiration in a child with altered mental status.

A full-term, 6-week-old baby is brought to the family physician for routine follow-up. At birth her weight, height, and head circumference were at the 50th percentile; she is now at the 5th, 10th, and 25th percentiles, respectively. She and her twin sister are exclusively breastfed; their mother has maintained a rigid every-four-hour feeding schedule since birth. Physical exam reveals a thin but otherwise healthy infant. What is the most likely cause of this infant's failure to thrive? Single Choice Answer: Please select one answer. A Congenital heart disease B Inadequate caloric intake C Cystic fibrosis D Milk protein allergy E Inborn error of metabolism

B has been selected by the expert. A. Congenital heart disease is incorrect for two reasons. First, CHD is a less common cause of FTT than option B. Second, CHD severe enough to cause FTT would present with associated signs (such as murmur, cyanosis, or hepatomegaly) and symptoms (such as easy fatigability). B. Inadequate caloric intake is correct because it is the most common cause of FTT. Furthermore, we have reason to believe that this particular infant is not receiving adequate calories. Breastfed infants should eat every two to three hours until 3 months of age; this infant eats every four hours and shares the milk supply with her twin. Twins can thrive on exclusive breastfeeding, but it requires that the mother consume extra fluids and calories and ensure that the twins both get enough to eat. C. Cystic fibrosis is incorrect because it is a less common cause of FTT than option B. CF condition affects fewer than 1 in 3000 Caucasians and is much less common in people of other ethnicities. Furthermore, malabsorption due to CF usually results in diarrhea, steatorrhea, and constant hunger. Respiratory conditions such as chronic cough or recurrent pneumonias may be present as well. D. Milk protein allergy is incorrect because, while it is fairly common (frequency of 2 to 3%), this infant has no symptoms. The most common symptoms include diarrhea, vomiting, abdominal pain, and allergic reactions ranging from urticaria to anaphylaxis. E. An inborn error of metabolism is incorrect because this category of conditions is quite rare and definitely less common than option B. Furthermore, the infant would likely have other symptoms (such as lethargy, vomiting, etc.).

A 12-year-old girl presents to her PMD complaining of a headache of gradual onset x 3 hours, non-provoked and described as a "big rubber band around my whole head" and a 5 out of 10 on the pain scale. The pain is not throbbing, and there is no associated photophobia, nausea or vomiting. The patient is afebrile, and there are no neurologic deficits during physical exam. Her mother states her pain is typically relieved with ibuprofen, but her mother is concerned that patient may have migraines because she has a few headaches every month after school. The child is otherwise healthy. What is the most likely cause of this girl's headaches? Single Choice Answer: Please select one answer. A Migraine B Tension-type headache C Brain tumor D Sinusitis E Pseudotumor cerebri

B has been selected by the expert. A. Migraines are typically described as having "throbbing"-type pain and may also be associated with nausea and/or vomiting with photophobia and phonophobia. Pain is often described at 10/10 and is debilitating. B. Tension-type headaches are often bilateral and involve the forehead, temporal areas, or back of the head. Tenderness of the posterior muscles of the neck may also be present. They should be responsive to NSAIDs. Stress can give rise to a tension headache, and this is consistent with this patient developing headaches after school. C. A brain tumor is unlikely to present without focal neurological findings, and the short duration of symptoms do not warrant further workup for this. In addition, the headache is less likely to be relieved with NSAIDs and would not occur only a few times a month. Headaches of new onset that do not go away and are worse in the mornings and associated with vomiting and personality changes would be most concerning for a brain tumor etiology. D. The patient is otherwise healthy and did not complain of any nasal stuffiness, rhinorrhea, or sore throat with postnasal drip, making sinusitis unlikely. E. Pseudotumor cerebri presents as symptoms of increased intracranial pressure—headache, nausea, blurred vision, diplopia, photophobia, and tinnitus—that occur in the absence of a intracranial mass. CT/MRI will be negative, but opening pressures are elevated on lumbar puncture. It typically occurs in obese women of childbearing age, with the median age of diagnosis around 30 years. While the cause is unknown, other risk factors include OCPs, growth hormone, excess vitamin A, and discontinuing steroids. This girl is probably too young to be given this diagnosis, and tension headache is more likely given the lack of symptoms of intracranial pressure.

Johnny is brought to your office by his parents for his 18-month well child check. His family recently moved into town, and this is the first time you are seeing him. Per mom, he was born at 32 weeks, and the details of his neonatal course are not clear. Parents share that he is not walking and they are very concerned. Johnny has met his social and language developmental milestones. Physical exam reveals spasticity, exaggerated deep tendon reflexes, and clonus in both of his lower extremities. An MRI of the brain is ordered, and the radiologist reports findings of periventricular leukomalacia. What is the most likely diagnosis? Single Choice Answer: Please select one answer. A Niemann-Pick disease B Spastic diplegia C Athetoid cerebral palsy D Autism spectrum disorder

B has been selected by the expert. A. Neimann-Pick disease is a neurodegenerative disease that can present in children between the ages of 6 months and 2 years. It causes global delays as well as regression of milestones. Other signs and symptoms include hepatosplenomegaly, interstitial lung disease, and a macular cherry red spot. B. Spastic diplegia is a form of cerebral palsy, a non-progressive static encephalopathy characterized by delays in motor development. It may be associated with periventricular white matter abnormalities that are thought to be due to ischemia. These changes can be visualized on MRI. In spastic diplegia, the motor abnormalities are often greater in the legs than in the arms. C. Athetoid cerebral palsy involves motor deficits of the entire body, not just the legs. It is often caused by perinatal asphyxia and kernicterus, both of which damage the basal ganglia, cerebellum, and/or thalamus. D. Children with autism present with delays in language and social interaction, rather than with motor delays. Motor development is typically normal.

A 4-year-old male with a history of Down syndrome and no other medical problems is brought to his pediatrician's office by his mother for increasing fatigue, intermittent fever, and decreased appetite for one week. On exam you note conjunctival pallor and hepatosplenomegaly. What is the best next step in the management of this patient? Single Choice Answer: Please select one answer. A Advise the mother to use acetaminophen as needed for fever and discharge home with strict return precautions. B Send CBC and peripheral smear C Send patient for chest x-ray D Send TSH E Start the patient on a 7-day course of amoxicillin and send home with strict return precautions.

B has been selected by the expert. A. This choice is incorrect because the patient is likely presenting with signs and symptoms of acute leukemia in a patient at increased risk for childhood malignancy (specifically ALL). B. This choice is correct because fatigue, decreased appetite, anemia, and HSM are all clinical signs and symptoms that may be associated with acute leukemia, for which patients with Down syndrome are at increased risk. A CBC would be helpful both for assessing the white blood cell count as well as the degree of anemia. A peripheral smear would be indicated to further evaluate for leukemia. C. This choice is incorrect because the patient is not presenting with any acute respiratory symptoms to warrant immediate chest x-ray. D. This choice is incorrect because, although Down syndrome may be associated with hypothyroidism, the patient's fever and physical exam findings are suggestive of a separate process. E. This choice is incorrect because the patient is presenting with signs and symptoms concerning for acute leukemia. Although infection may complicate leukemia, an outpatient course of amoxicillin would not be appropriate therapy at this time.

"What if, on Nick's neurologic examination today, you'd found that he had papilledema, paresis and sensory loss over the right leg with 4+ reflexes at the patella and ankle also on the right—and that the rest of his neurologic examination was normal. With those findings, where would you think the lesion might be?" Multiple Choice Answer: A Supratentorial, on the right side of the brain B Supratentorial, on the left side of the brain C Infratentorial on the right side of the brain D Infratentorial on the left side of the brain E Brain stem

B has been selected by the expert. Distinguishing Neurologic Examination Findings Infratentorial lesions usually present with cerebellar signs and signs of raised intracranial pressure (ICP). Cerebellar hemispheric lesions can cause changes in muscle tone and DTRs, but usually lead to hypotonia and hyporeflexia. Supratentorial lesions more commonly lead to focal motor and sensory abnormalities on the side opposite to the lesion. Brain stem tumors will often be associated with cranial nerve and gaze palsies.

A 7-year-old boy is brought by ambulance to the ED with altered consciousness. The EMT said he found the boy in a pool of vomit. He is unable to answer questions coherently and he is alone. Physical exam findings indicate dry mucous membranes, tachypnea, tachycardia, and moaning on palpation of the abdomen. His physical exam is otherwise normal, including a normal blood pressure. What is the most likely cause of his condition? A Appendicitis B DKA C Narcotic overdose D Non-accidental trauma E Sepsis

B has been selected by the expert. B. DKA typically presents with altered mentation, vomiting, dehydration, and abdominal pain. The history will yield polydipsia and polyuria during the days preceding DKA. Metabolic acidosis causes tachypnea as the body tries to blow off CO2 through a compensatory respiratory alkalosis. A. Appendicitis would rarely present with altered consciousness. The usual history with appendicitis is onset of periumbilical pain that persists over hours, migrating to the right lower quadrant. Vomiting could be present, and tachycardia may be present due to pain or dehydration, but altered mental status would be unusual. On physical exam, peritoneal signs may be present as well as psoas, obturator, or Rovsing's sign. C. Although he may have access to narcotics, his presentation does not fit well with this choice. Signs of narcotic overdose include pinpoint pupils, depressed respiratory rate, and altered consciousness. His tachypnea and lack of pinpoint pupils argue against this choice. D. Lack of fractures, bruises, or burns argues against this choice. Trauma resulting in increased intracranial pressure may result in hypertension, bradycardia, and disordered breathing. E. Sepsis can present with altered mental status. This child's presentation is less consistent with sepsis given that he doesn't have fever or other vital sign changes consistent with sepsis syndrome (temperature > 38.5°C or < 36°C, hypotension, along with warm, dry extremities).

A 9-year-old male is brought to the ED in a coma secondary to diabetic ketoacidosis. Which of the following laboratory results would NOT likely be found in this patient? A Anion gap of 20 mEq/L B Potassium of 3.3 mEq/L C Venous pH of 7.1 D Sodium of 132 mEq/L E Creatinine of 1.0 mEq/L

B has been selected by the expert. B. In diabetic ketoacidosis, the acidosis and lack of insulin cause potassium to leave cells and enter the serum, causing an elevated serum potassium level. However, as the DKA is corrected and insulin is administered, the potassium will re-enter the cells, causing a decreased serum potassium level, so potassium levels should be monitored closely when therapy is initiated. A. DKA causes a metabolic acidosis from the elevated level of ketones. The elevated level of ketoacids and lactic acid requires buffering by bicarbonate, thus leading to an increased anion gap. C. The pH would be low due to the metabolic acidosis caused by the elevated level of ketones. D. Hyponatremia is seen in DKA because the hyperosmolarity of the intravascular space from the increased glucose levels causes osmotic movement into the extracellular space. Additionally, there is increased sodium loss from the kidneys. A corrected sodium level should be calculated to adjust for the hyperglycemia, using the following formula: corrected sodium = [{(measured glucose - 100)/100} x 1.6] + measured sodium. E. Patients with DKA are often dehydrated when they present to the ED. This causes a prerenal azotemia, which presents as an elevated creatinine level.

An 11-year old boy presents to clinic with wheezing. Mom states that in the past he has used inhaled albuterol and it has helped with wheezing and shortness of breath. On further history you find out that the patient experiences shortness of breath three times a week and is awakened at night by these symptoms once a week. What is the most appropriate outpatient therapy? A Only rescue inhaler PRN B Low dose inhaled corticosteroids C Medium dose inhaled corticosteroids and course of oral corticosteroids D Medium dose inhaled corticosteroids, LABA, and course of oral corticosteroids E Course of oral corticosteroids

B has been selected by the expert. A. Rescue inhaler (a short-acting beta agonist) i PRN is incorrect because this treatment is indicated in patients with intermittent asthma and have symptoms fewer than two days a week or two nights a month B. Low dose inhaled corticosteroid is correct because this patient has mild persistent asthma. His symptoms occur 3-6 days/week and 3-4 nights/month. C. Medium dose inhaled corticosteroids with a course of oral corticosteroids is incorrect, because it would be indicated in a patient with moderate persistent asthma when symptoms occur daily and more than one night per week. D. Medium dose inhaled corticosteroids, LABA, and oral corticosteroids is incorrect because this patient does not have severe persistent asthma. E. A course of oral corticosteroids alone is incorrect. Asthma needs to be managed long term to prevent exacerbations. An inhaled corticosteroid is indicated.

Susie is a 3-year-old girl brought into the clinic by her mother because she has a gradually worsening cough and she has been having trouble breathing. Her mother says Susie sounds like she is barking when she coughs. Susie is up to date with her vaccinations. Susie's mom always watches her when she's playing. On physical exam, you note that Susie has inspiratory stridor. She does not have wheezing, there are no retractions, and she has symmetrical breath sounds. No pseudomembranes are appreciated on physical exam. What is Susie's most likely diagnosis? A Epiglottis B Croup (laryngotracheobronchitis) C Pertussis D Pneumonia E Foreign body aspiration

B. Croup or laryngotracheobronchitis is due to a viral infection (Parainfluenza type 1). It is most common in the winter, and often occurs in children age 2 to 5 years. Croup can lead to non-specific URI symptoms with some degree of airway obstruction. A barky or seal-like cough and inspiratory stridor (which should be differentiated from expiratory wheezes) is common in croup. A. Epiglottitis is a life-threatening emergency caused by an infection with H. influenzae type B. It is less common now with the advent of Hib vaccine, but in rare cases can occur due to staphylococcal or streptococcal infections. It most often occurs in children ages 2 to 5 years. Children with epiglottitis present with fever, stridor, drooling, dysphonia, dysphagia, and respiratory distress. They frequently appear toxic and sit in the "sniffing position" (sitting, leaning forward, neck hyperextended, chin protruding). A "thumb sign" (thickened epiglottis and aryepiglottic folds) appears on films. Susie is not exhibiting any of these characteristic symptoms and she is up to date with vaccines, making epiglottitis a less likely diagnosis for her cough. C. Pertussis occurs in three phases: The catarrhal stage lasts one to two weeks when children present with URI symptoms. A paroxysmal stage follows and lasts four to six weeks. During this phase, children have repetitive forceful coughing with massive inspiratory effort ("whoops"). Finally, during the convalescent stage, children present with continued cough that may last up to three months. The acellular pertussis vaccine protects against pertussis, and, as Suzie is up to date with her immunizations, this diagnosis is less likely. Her barking cough is more suggestive of croup than the "whooping" cough of pertussis. D. While retractions and cough are present with pneumonia, asymmetric breath sounds and tachypnea would be more specific for a diagnosis of pneumonia. E. Symmetrical breath sounds and the gradual onset of Susie's cough make foreign body aspiration less likely. One might also expect focal wheezing and tachypnea with foreign body aspiration.

A 10-month-old infant is brought to the Peds ED by her parents, who say she has been coughing persistently for the last three hours. The parents were watching a movie at home when they first noticed their daughter coughing. Patient is a vaccinated, well-nourished infant in moderate distress with retractions, nasal flaring, and grunting. On auscultation, you immediately notice diminished breath sounds in the right lung with normal breath sounds on the left. What other associated physical exam finding do you expect to hear? A Stridor B Asymmetric breath sounds and wheezing C Rhonchi D Crackles E Bronchial breath sounds

B. This infant is in respiratory distress from foreign body aspiration, consistent with the history of acute onset of distress and asymmetric breath sounds. Common foreign bodies include peanuts, popcorn, grapes, hard candy and hot dogs. Respiratory distress from foreign body aspiration is usually accompanied by asymmetric breath sounds and wheezes on auscultation. A. Stridor is due to airway narrowing above the thoracic inlet and can be seen in URI such as epiglottitis. You might expect stridor in laryngomalacia, but this would not have suddenly appeared at 10 months of age. Also, epiglottitis is more rare now in a vaccinated child, and you wouldn't expect to hear unilaterally diminished breath sounds. C. Rhonchi are coarse, low-pitched, rattling sounds due to secretions and airway narrowing and are typically heard in the setting of bronchitis or pneumonia. D. Crackles are due to fluid in alveoli or opening and closing of stiff alveoli, not consistent with a foreign body aspiration. You would expect this in either pneumonia or CHF from pulmonary edema, both of which are unlikely in this patient given the abrupt onset and lack of history of cardiac problems. E. Bronchial breath sounds are hollow-sounding and caused by air moving through areas of consolidated lung, such as in the setting of pneumonia.

When evaluating a person with a possible stroke, when is it acceptable to administer t-PA? A If onset of neurologic symptoms was <6 hrs prior to presentation B if emergent CT head shows a hemorrhagic stroke C If onset of neurologic symptoms was <3 hours prior to presentation D If emergent CT head shows early infarction

C has been selected by the expert. The correct answer is C. When assessing a patient who presents acutely with symptoms suggestive of a stroke, one possible treatment option is using recombinant tissue plasminogen activator. When used in the appropriate patient, this medication can help minimize the damage done related to suffering an acute ischemic injury to the brain. However, it is only acceptable to administer t-PA in select situations. These include when onset of neurologic symptoms has been within three hours of the onset of symptoms, emergent CT head does not show an intracranial bleed, an early acute infarct, or a brain mass.

A 7-year-old boy with a past medical history of headaches presents with increased frequency and severity of headaches along with new onset vomiting. When the patient was walking into the room, he had a wide stance and nearly tripped twice. Which of the following is the most appropriate next step? Single Choice Answer: Please select one answer. A Daily headache diary B Computed tomography C Magnetic resonance imaging D Lumbar puncture E Intraventricular pressure monitoring

C has been selected by the expert. A. A daily headache diary is a valuable clinical tool. It should include: characterization, duration, location, associated symptoms, recent activity, aggravating, and alleviating factors. This particular case includes multiple concerning symptoms and requires a more in-depth workup. B. CT imaging is a faster and more convenient than an MRI and is very valuable in ruling out intracranial hemorrhage. However, it does not visualize the posterior fossa as well as MRI and would not be the modality of choice if MRI is available. C. MRI is more expensive and less readily available than CT imaging. It also frequently requires sedation in pediatric patients. However, it provides the best detail of the posterior fossa, which is the most common location of pediatric brain tumors. D. Lumbar puncture can be very helpful in the diagnosis of meningitis, but would not be appropriate in this case. This patient may have increased intracranial pressure, and lumbar puncture is contraindicated in patients with signs of increased intracranial pressure because it may lead to brain herniation. E. Intraventricular pressure monitoring involves the placement of an intraventricular catheter, which measures pressure inside the skull and sends measurements to a recording device. This option may reveal increased intracranial pressure, but would not be helpful in the diagnosis. A brain MRI would be more appropriate.

You are called to the delivery of an infant boy experiencing fetal distress. After a vaginal delivery with vacuum assist, the infant cries spontaneously but remains acrocyanotic, despite supplemental oxygen delivered by mask. The neonate is hypotonic and moves his extremities only in response to noxious stimuli. Physical exam reveals an open mouth with a protruding tongue, upslanting palpebral fissures, low-set ears, and a transverse crease across both palms. You immediately recognize this syndrome, and your attending asks you what is the most common cardiac defect in these patients? Single Choice Answer: Please select one answer. A Aortic insufficiency B Coarctation of the aorta C Endocardial cushion defects D Patent ductus arteriosus E Conduction pathway defects

C has been selected by the expert. A. Aortic insufficiency is associated with Marfan syndrome. Aortic dissections are also seen in this condition and can be lethal. B. Coarctation of the aorta is seen in 35% of patients with Turner syndrome. C. Endocardial cushion defects. This patient has physical signs of Down syndrome. Approximately 50% of children with Down syndrome are born with endocardial cushion defects, such as ventricular septal defect, atrial septal defect, or complete atrioventricular canal defect. D. Patent ductus arteriosus is usually seen in infants with prematurity, sepsis, metabolic acidosis, or pulmonary defects not associated with congenital syndromes. E. Conduction pathway defects are not characteristic of patients with Down syndrome but may be seen in infants born to mothers with lupus (heart block).

A 15-month-old is able to stand on his own, walk backward, and throw objects underhand. He is unable to draw/scribble or grasp markers. He can say one syllable words "ma" and "da" but his words are unintelligible. He holds a sippy cup with help. He does come when called, plays with a ball and waves bye-bye by imitating his parents. Does this baby have developmental delay, and, if so, within which domains? Single Choice Answer: Please select one answer. A Normal variant B Gross motor and fine motor delay C Fine motor and language delay D Language delay and gross motor E Social and language delay

C has been selected by the expert. A. Incorrect. A 15-month-old child should be more developed in language and fine motor skills B. Incorrect. This child is able to walk backward and throw objects without difficulty. However, he does exhibit some delay with fine motor skills with his inability to scribble or hold a sippy cup on his own. C. Correct. A 15-month-old child should be able to scribble, use a cup, and/or stack 2 blocks. His word count should be three to six words at this stage of development. D. Incorrect. Although this child does have some evidence of language delay, his gross motor skills are at a normal stage of development. E. Incorrect. Socially this child is within normal developmental range since he comes when called and he imitates others.

Kenny is a 12 year-old male who comes to your clinic with a chief complaint of crampy abdominal pain. His mother tells you that sometimes he wakes up from sleep due to the pain. He also has diarrhea that sometimes has blood in it. When asked about stressors in his life, his mother sighs and tells you that she is recently divorced and had to move Kenny to a new school. On physical exam, he appears small for his age. Abdomen is soft, non-distended, but tender to palpation at the RUQ. On rectal exam, you note anal skin tags and an anal fistula. Skin exam shows red tender nodules on his shins. Labs show a microcytic anemia. What is the next best step in management? A Reassure and refer to psychiatry B Start omeprazole and antibiotics C Colonoscopy D Start mesalamine E Obtain IgA endomysial antibody and IgA anti-tissue transglutaminase antibody

C has been selected by the expert. A. Reassure and refer to psychiatry is incorrect. This is the right management for functional abdominal pain, which is the most common cause of abdominal pain in this age group. However, bloody diarrhea cannot be explained by functional abdominal pain and suggests a more serious diagnosis. B. Start omeprazole and antibiotics is incorrect. This is the correct management of PUD caused by H. pylori. PUD cannot account for the perianal disease, diarrhea, or erythema nodosum. C. Colonoscopy is the best answer. Kenny likely has IBD (Crohn's disease or ulcerative colitis [UC]). He has crampy abdominal pain and intermittently bloody diarrhea, crampy abdominal pain (that wakes him up at night), perianal disease, and erythema nodosum. He also has microcytic anemia, likely from chronic blood loss. Colonoscopy with biopsies will allow you to diagnose Crohn's disease (or UC) prior to treating it. The diagnosis begins with a colonoscopy to obtain tissue biopsies as well as blood tests (p-ANCA, ASCA). D. Start mesalamine is incorrect. Mesalamine is the first-line treatment for Crohn's disease. However, you have not yet established a diagnosis. E. Obtain IgA endomysial antibody and IgA anti-tissue transglutaminase antibody is incorrect. This is the right diagnostic test for celiac disease. However, celiac disease would not present with blood in the stool or other extraintestinal signs. Celiac can be associated with dermatitis herpetiformis, an itchy, papulovesicular rash.

A 39-year-old G2P1 woman with a pre-pubertal 10-year-old boy with intellectual disability comes to the clinic for information on prenatal screening. The 10-year-old boy was born with large ears and long face but no other congenital malformations. The mother is worried that she will have a second child with similar problems. If she were to have a second male child with developmental impairment, what would be the most likely reason if maternal serum testing and fetal ultrasound were both normal? Single Choice Answer: Please select one answer. A Down syndrome B Trisomy 13 C Fragile X syndrome D Turner syndrome E Klinefelter syndrome

C has been selected by the expert. A. Risk of Down syndrome is increased with advance maternal age, but prenatal screening would show abnormal maternal serum values with or without abnormal fetal ultrasound findings. B. Patients with Trisomy 13 would not have large ears and long face, but would present with severe impairments at birth. C. Fragile X syndrome is the most common familiar cause of developmental impairment and is due to an abnormal number of trinucleotide repeats. These children present with large ears, long face and mandible and, after puberty, large testicles. D. Turner syndrome is not correct given the child's male sex. Individuals with Turner syndrome have normal IQ. E. Patients with Klinefelter syndrome have IQs that vary around the low-normal range, but typically have no stereotypical physical features other than low muscular bulk due to decreased testosterone levels.

A 12-day-old baby girl is brought to the ED by her foster mother due to fussiness and tactile fevers. The baby's teenage biological mother did not receive prenatal care and delivered her baby at home. On further questioning, you find out that the patient has had only two wet diapers per day and two loose green stools per day. On exam, the patient is irritable and her anterior fontanelle is tense. Which of the following diagnoses are of emergent concern at this time? A Down syndrome B Fetal alcohol syndrome C Group B strep sepsis/meningitis D Meconium ileus E Poor weight gain

C has been selected by the expert. A. This choice is incorrect, because although the patient is likely to have missed newborn screening for Down syndrome, her most emergent concern is her fever and possible sepsis. B. This choice is incorrect. Although the patient is at risk for fetal alcohol syndrome (since her mother did not have prenatal care and may not have had counseling regarding abstinence from alcohol), her most urgent concern is her fever in a less than 1-month-old infant. C. This choice is correct. The patient's mother did not have prenatal care and likely did not have screening for group B strep during pregnancy. She also delivered at home and would not have had access to antibiotics during delivery. Group B strep is a common and serious cause of sepsis and meningitis in newborns. D. This choice is incorrect because the patient reportedly has passed stool since delivery. E. This choice is incorrect. Infants may lose weight during the first few days of life, but are expected to regain their birth weight by 2 weeks of age.

A 21-year-old female with no significant past medical history experienced an inversion-type injury to her right ankle while playing soccer a day prior to presentation. She remembers immediate pain and swelling but was able to weight bear and limp off the field. She has noticed some significant swelling which is mostly still present. She has been icing the ankle since the injury as her coach recommended. Pain is still present near the lateral malleolus. Physical exam reveals that the lateral right ankle is edematous with purplish bruising, bilateral pulses are good and the patient had good sensation and motor function in both feet about equally. Palpation of the posterior edge of the lateral malleolus elicits significant pain from the patient. What is the next best step in the management of this patient? A Reassurance B Immobilize with cast or splint C X-ray imaging of right ankle D Emergent fasciotomy E Rest, ice, Ibuprofen, compression and elevation and re-evaluate in one week

C has been selected by the expert. Based on the Ottawa Ankle Rules, tenderness of the lower 6cms of the posterior lateral malleolus may predict fracture and justifies X-ray imaging of the ankle. Tenderness of the lower anterior lateral malleolus, on the other hand, is very common in ligamentous injury such that X-rays are not warranted. Because of the possibility that a fracture exists, the other treatment options are not correct in this scenario.

Ms. Martinez, 74-years-old, is brought to your family practice by her husband. He is concerned because his wife, who used to take pride in keeping the house in good shape, has stopped cleaning and caring for her appearance. She recently went shopping and left the stove on. She frequently misplaces her car keys and checkbook. Ms. Martinez reports trouble concentrating and does not have much energy. She has a poor appetite and no longer enjoys knitting. Her mini-mental status exam is 28/30. Physical Exam: General Appearance: well-nourished, appears stated age, HEENT[LW1] : pupils equally round and reactive to light, moist mucus membranes, Chest: normal S1 and S2, no murmurs, Lungs: clear to auscultation bilaterally, Abdomen: soft, non-tender, non-distended, Extremities: no cyanosis, no clubbing. Neurologic: no focal deficits Psychiatric: flat affect. Labs: TSH: 2, WBC: 6.0, HgbA1c: 4.5. Which of the following is the most likely diagnosis? Single Choice Answer: Please select one answer. A Normal bereavement B Dementia C Depression D Pseudo dementia E Delirium

C has been selected by the expert. The correct answer is C. Ms. Martinez has a normal mini mental status exam, her exam findings are normal with the exception of her affect. This suggests a mood disorder. As there is nothing in the history to suggest a recent loss, bereavement is not likely. History is not suggestive of delirium which would present with more fluctuations in symptoms and alteration in cognition

Ms. Michaels is an 80-year-old female with a past medical history of shingles. She comes to your office accompanied by her daughter Jennifer who reports that her mother is forgetting things. Jennifer explains that her mother will ask the same question several times throughout the day although it was answered. Ms. Michaels also gets confused easily and is more passive than usual. Her memory problem was noticed two years ago after she forgot to pay her bills on multiple occasions. Jennifer now pays her mothers' bills and cleans and cooks for her. Ms. Michaels' vital signs are T=99.2, BP= 118/70, HR=80, RR=12. Her physical exam is significant for bilateral osteoarthritis hand deformities. CT head shows mild atrophy of the hippocampus. Her MMSE is 20. The patient's diagnosis is most likely associated with? Single Choice Answer: Please select one answer. A Lewy bodies B Caudate nucleus atrophy C Plaque formation D Prion protein E Vascular disease

C has been selected by the expert. The correct answer is C. Ms. Michaels symptoms and CT results are consistent with Alzheimer's Disease as are plaque formations. Lewy bodies are abnormal aggregates of protein that develop inside nerve cells in Parkinson's disease. Huntington's disease presents with caudate nucleus atrophy. Prion proteins are seen in transmissible spongiform encephalopathies

A 41-year-old male with no significant past medical history is brought to the ED after falling to the ground in the middle of a pick-up basketball game with friends. He did not lose consciousness nor hit his head when he fell. As he landed on the ball of his foot after having taken a shot, he recalls hearing a popping sound followed by immediate pain in the posterior right ankle. On physical exam, the posterior right ankle is edematous and palpation is tender. He is unable to plantarflex his right foot. What is the most likely diagnosis of his current condition? A Ankle ligament sprain B Calcaneal fracture C Achilles tendon rupture D Ankle tendonitis E Ankle arthritis

C has been selected by the expert. This is a classical description of an acute rupture of the achilles tendon. Middle-aged males are more commonly affected than other groups. The mechanism does not describe inversion injury making an ankle ligament sprain less likely and there is no direct trauma making fracture less likely. There is no history of overuse or chronicity making arthritis and tendonitis less likely.

A 5-year-old boy is noted to have a grade II systolic murmur and a widely split S2 murmur on cardiac exam. His vital signs are stable and he has been asymptomatic. Which of the following statement is accurate regarding this patient's presentation and likely condition? Please select one answer. A No further work-up for a presumed venous hum B Chest x-ray, ECG, and echocardiogram would be indicated as next steps to work up a presumed ventricular septal defect C This patient's murmur is caused by flow through the pulmonary outflow tract and should be evaluated D The patient should be scheduled now for cardiac catheterization

C has been selected by the expert. C. Correct. This patient's murmur is likely caused by an atrial septal defect, which causes flow of additional blood through the pulmonary outflow tract and should be evaluated. A. Incorrect. A benign venous hum would not be expected to present with a widely split second heart sound. B. Incorrect. The first steps in the evaluation of presumed structural heart disease would include a chest x-ray, ECG, and echocardiogram. However, the clinical presentation is not consistent with a ventricular septal defect, which usually presents with a holosystolic murmur and a second heart sound that splits variably with respiration. D. Cardiac catheterization might be indicated to better define this patient's anatomy, and the pressures within each cardiac chamber. However, it would not be indicated to schedule this before a preliminary work-up, including a chest x-ray, ECG, and echocardiogram.

A 9-year-old male presents to the ED in an ambulance after he was found unconscious at a local playground. In the ED he is arousable but extremely obtunded. He is able to minimally verbalize that his head hurts and his stomach feels uncomfortable. He states the pain is constant and non-radiating. He vomits clear liquid twice over the course of 30 minutes. Vital signs are as follows: T 37.6 C, P 66 bpm, BP 155/80 mm Hg, RR 18 bpm. You further notice that his breathing is irregular with brief episodes of apnea. On physical exam you are unable to reproduce the abdominal pain and there is no rebound tenderness or guarding. The rest of the physical exam is unremarkable. What is the most likely diagnosis? A DKA B Appendicitis C Intracranial hemorrhage D Gastroenteritis E Small bowel obstruction

C has been selected by the expert. C. This is the correct choice. Increased ICP can be secondary to epidural or subdural hemorrhage. It is possible the patient may have fallen while playing in the playground. Increased ICP can present as the classic Cushing's triad: hypertension, inappropriate slowing of the heart rate, and irregular respirations (Cheyne-Stokes respiration). A further complication of increased ICP is epigastric discomfort. This is caused by the elevated ICP causing vagal stimulation, resulting in the secretion of gastric acid. Lastly, the patient's headache and non-bilious vomiting can also be ascribed to the increased ICP. A. Choice A is incorrect, because in a patient with DKA one would expect increased adrenergic tone leading to tachycardia, not inappropriate slowing of the HR. Secondly, the patient's breathing pattern is more consistent with Cheyne-Stokes respirations, not Kussmaul breathing. Kussmaul breathing is typically characterized by deep breaths that may be rapid, normal or slow in rate without periods of apnea, often associated with metabolic acidosis. Lastly, one would expect signs or symptoms consistent with dehydration such as polyuria, polydipsia, decreased skin turgor, or skin tenting. However, the altered mental status, vomiting, headache, and abdominal pain could be seen in DKA. B. Choice B is incorrect because the lack of fever, inability to reproduce the abdominal pain in the RLQ, and the severely altered mental status argue against the diagnosis. In appendicitis, children will often complain of a migratory pain that beings around the periumbilical region and migrates to the RLQ. Patients often complain of rebound tenderness and demonstrate guarding as well. D. Choice D is incorrect because gastroenteritis usually presents with fever, colicky abdominal pain, and diarrhea. It would also be atypical for the patient's mental status to be so adversely affected by gastroenteritis. More than 95% of gastroenteritis hospitalizations occur in children younger than 5 years, with the peak incidence between 3 and 24 months of age. The incidence tends to peak in winter. There can be both viral and bacterial causes for gastroenteritis. Classically, viral gastroenteritis will present with diarrhea in which the stool lacks blood or mucus. Bacterial gastroenteritis often causes diarrhea with gross blood or mucus present in the stool. E. Choice E is incorrect because a small bowel obstruction usually presents with bilious vomiting, abdominal distention, inability to pass flatulence, and moderate-to-severe abdominal pain. The pain is often paroxysmal, coming and going in 4 to 5 minute intervals. The patient's lack of a fever, however, is consistent with a GI obstruction. The most common causes of a small bowel obstruction are adhesions from a previous surgery or a hernia.

A 9-year-old male presents to your clinic with discoloration under his eyes, persistent cough, and skin rashes. He is found to have wheezing on physical exam and increased lung volume bilaterally on chest x-ray. He has struggled with these complaints over the past three years but recently his symptoms have gotten worse, affecting him every other day. He is afebrile. He is found to have wheezing on physical exam and increased lung volume bilaterally on chest x-ray. What would be the most appropriate treatment for him? A Oral antibiotics B Short-acting beta agonist PRN C Short-acting beta agonist PRN with low-dose inhaled corticosteroid D Short-acting beta agonist PRN with medium-dose inhaled corticosteroid E Long-acting beta agonist

C has been selected by the expert. A. The patient's presentation is more consistent with asthma than an infection. The patient has had these complaints for the last few years. His skin rashes and lower eyelid darkening are consistent with allergic processes (atopy), which are associated with asthma. Furthermore, increased lung volumes bilaterally and persistent cough without fever also suggest asthma, thus antibiotics would not be appropriate. B. Cough and wheezing that occur intermittently (< 2 days/week) are consistent with intermittent asthma, which is treated with short-acting beta agonist PRN. C. Persistent cough and wheezing that affect the patient every other day (3-4 days with symptoms/week) are consistent with mild persistent asthma, which is appropriately treated with short-acting beta agonist PRN and low dose inhaled corticosteroid. The swelling under the eyes (allergic "shiners") and skin rash are other signs of atopy, as mentioned above. D. Short-acting beta agonist PRN with medium dose inhaled corticosteroid is the preferred treatment for moderate or severe persistent asthma, which corresponds to daily symptoms or symptoms throughout the day. E. The use of a long-acting beta agonist is reserved for severe persistent asthma, which corresponds to symptoms throughout the day.

A woman brings her 8-year-old son to the pediatrician after witnessing him stare blankly into the distance at dinner the previous week. He was unresponsive to her calling his name or any other stimuli, and it lasted for about 10 or 20 seconds. His teacher reports he does seem to daydream often in class but is able to keep up with schoolwork and excels in his studies. She doesn't note him being disruptive or impulsive in class. His mother is concerned about these blank stares and unresponsive episodes. Which of the following is the most likely diagnosis? A Generalized tonic-clonic seizure B Atonic seizure C Absence seizure D Simple partial seizure E Complex partial seizure

C has been selected by the expert. A. Generalized tonic-clonic seizure is the most common type of seizure seen in children. These seizures begin acutely with tonic (rigid) stiffening of all extremities and upward deviation of the eyes. The tonic phase is followed by clonic jerks of all the extremities. The patient then loses consciousness and may have urinary incontinence. The patient in this vignette did not have movement of his extremities and did not have urinary incontinence. B. Atonic seizure, also called drop seizures or drop attacks, consists of loss of motor tone, where patients have a brief lapse in muscle tone. These seizures can occur while walking, standing, or sitting, and have a characteristic head drop (the neck muscles releasing). These seizures last about fifteen seconds. The patient in the vignette did not have symptoms of an atonic seizure. C. The patient is having absence seizures. Absence seizures are characterized by loss of awareness of surroundings ("blank stare" or "in another world") and automatisms (e.g., eye-fluttering or lip-smacking). These patients do not lose consciousness or have loss of tone. Absence seizures should also be differentiated from ADHD, since children with ADHD also can be inattentive or seem to be daydreaming. However, since the patient in the vignette still does well in school and does not have other signs of ADHD, it is most likely an absence seizure. An EEG will confirm the diagnosis. D. Simple partial seizure is a seizure localized to a small region of the brain. Patients do not lose consciousness. Based on the region of the brain the seizure is occurring in, the patient will exhibit focal symptoms (e.g., altered hearing or smelling, labored speech, etc.). If patients have motor symptoms, it is usually localized to one extremity. E. Complex partial seizure is characterized by impairment of consciousness. These seizures are similar to simple partial seizures except patients lose consciousness.

A 10-year-old male comes to the clinic with a chief complaint of progressive cough for two weeks that began gradually. His cough is described as productive and wet with whitish sputum. His mother denies throat pain, vomiting, and diarrhea in his review of systems. His mother reports that he has been febrile up to 101.5°F daily. She thinks he is fatigued and has not eaten well in the past week. On exam, there is air passage throughout all lung fields, with crackles in the lower right lung field, but no other abnormal sounds. What would you likely find in your workup? A Response to inhaled beta-agonist B Hyperinflation in one lung field C Alevolar consolidation in the RLL D Positive PCR for pertussis E Fluffy bilateral infiltrates and a large heart on chest x-ray

C has been selected by the expert. A. Response to an inhaled beta agonist is a good test for RAD or asthma. Asthma is diagnosed clinically, usually in a school-aged child, with a history of recurrent wheezing. Associated findings might atopic stigmata, such as allergic rhinitis, food allergy, and atopic dermatitis. Without details in the history, asthma is a less likely diagnosis in this setting, and in the absence of wheezing, asthma is even less likely. B. This would be consistent with a foreign body aspiration which could produce cough and fever (if bacterial superinfection occurs). This diagnosis is usually considered in younger children. Constitutional symptoms (fatigue, decreased eating) make pneumonia a better diagnosis in this clinical setting. C. Pneumonia is the most likely cause for his symptoms and a chest x-ray would be a great confirmation of your suspected diagnosis. Eliciting a complete history might reveal history of an upper respiratory infection. Localization of crackles (discontinuous inspiratory sounds) to one lobe makes pneumonia more likely. D. Pertussis can produce a lengthy cough illness, but is not associated with fever or lung findings. E. Pulmonary edema due to CHF is a symmetrical process and less likely to present with a unilateral lung finding. Pulmonary edema should be suspected with crackles, but this clinical setting leaves pulmonary edema low on our differential.

Dr. Rodriquez, your family medicine preceptor is giving a talk about ways to prevent delirium in patients during their hospital stay. What did she most likely suggest? Single Choice Answer: Please select one answer. A Only allow the patients visitors to come at night B Give diazepam every six hours C Keep the patient in restraints D Keep the patients room quiet E Keep the patients room well lit

E has been selected by the expert. The correct answer is E. Stimulation such as a well lit room with activity has been shown to decrease delirium in hospitals. Limiting visitors and keeping the room quiet could increase the risk of delirium. Benzodiazepines such as diazepam can bring on delirium. Restraints would be a response

Casey's parents just learned that Casey's screening test for CF was abnormal. What other signs and symptoms might you expect this patient to develop? Single Choice Answer: Please select one answer. A Bloody stools B Precordial thrill C Reflux D Greasy stools E Diaphoresis during feeds

D has been selected by the expert. A. Bloody stools would be expected in a child who had gastroenteritis or intussusception, or other intestinal anomaly. You might also expect mucus in the stool in cases of gastroenteritis or intussusception. B. A precordial thrill would be indicative of a cardiac problem, but cardiac issues are not associated with CF. C. There is no association between gastresophageal reflux disease and CF. D. CF causes exocrine insufficiency and ultimately results in fat malabsorption. This causes frequent foul-smelling and greasy stools. E. Diaphoresis during feeds would raise concern for congestive heart failure. The classic associated signs include tachypnea, tachycardia, and hepatomegaly.

During the middle of dinner on your day off, you receive a call from one of your neighbors who remembers that you are a medical student. He is concerned about his 15-year-old daughter who had previously been in her usual state of health and has no significant past medical history. However, over the past 24 hours, his daughter suddenly spiked a fever of 103 F and has "not been herself," acting very lethargic and dazed at times. He also notes that she has been breathing heavily, not been able to eat or drink, and has not urinated over the past 12 hours. He wants your advice about whether she should be taken to the ED. Although you are fairly certain that the best course of action would be to take her to the ED, you contemplate the differential diagnosis of her presentation. Given the limited history, which of the following is highest on your differential? A Acute cystitis B Diabetic ketoacidosis C CNS tumor D Meningitis E Hypoglycemia

D has been selected by the expert. A. Choice A is incorrect because acute cystitis is an infection that is usually limited to the lower urinary tract. While acute cystitis can present with fevers and dysuria, it would likely not present with more systemic systems, like tachypnea, altered mental status, and decreased PO intake. Such a systemic presentation would be more characteristic of bacteremia secondary to pyelonephritis. B. Choice B is incorrect because diabetic ketoacidosis is usually associated with increased (not decreased) urine output secondary to the osmotic effect of ketones and glucose in the urine. Thus, while diabetic ketoacidosis should certainly be on the differential for causes of altered mental status and lethargy, it is not high on the differential given the presentation. C. Choice C is incorrect because a CNS tumor would likely not result in such an acute presentation of altered mental status. While a CNS tumor may result in tachypnea and lethargy depending upon the location of the tumor, the presentation would likely be a more slow and gradual progression. Furthermore, a patient with a CNS tumor would likely not present with such a high-grade fever, which places a CNS tumor lower on our differential. D. Choice D is correct because meningitis is the most likely etiology in our differential given the fever, altered mental status, decreased PO intake, and decreased urine output. While the incidence of meningitis has decreased in this patient's age range due to increased vaccinations against the most common causative organisms of meningitis, it still remains high on our differential given the presentation of this patient. In the ED, we would likely need to obtain a more thorough history and physical exam as well as blood cultures and lumbar puncture to establish the diagnosis of meningitis. E. Choice E is incorrect because hypoglycemia is unlikely to be source of such a high-grade fever (unless the patient had sepsis which resulted in high fever and in hypoglycemia). Furthermore, hypoglycemia would likely not affect urine output over such a significant period of time. While hypoglycemia should certainly be on the differential for causes of altered mental status, the presentation of fever and decreased urine output makes places it lower on our differential.

A 4-day-old male infant was born by vaginal delivery to a 35-year-old G1P1 who declined prenatal screening. The infant has mild hypotonia, epicanthal folds, upslanting palpebral fissures and a flat face. On physical exam, he is in no acute distress, has normal oxygen saturation and has a continuous murmur. Lymphocyte karyotyping showed a particular change in chromosome number. What genetic abnormality is most likely? Single Choice Answer: Please select one answer. A Trisomy 18 B 45 X0 karyotype C Trisomy 13 D Trisomy 21 E 47 XXY karyotype

D has been selected by the expert. A. Choice A is incorrect because this describes Edwards syndrome, caused by an extra chromosome 18. These infants present with severe mental retardation, micrognathia (small jaw), low-set ears, short neck, overlapping fingers (digits 2 and 5 overlap medially upon closure of fist), heart defects, renal malformations, limited hip abduction, and "rocker-bottom" feet (the plantar side of foot has a u-shaped curve like a rocking chair). B. Choice B is incorrect because this is characteristic of Turner syndrome. Patients with Turner syndrome are female (males demise) and present with neck lymphedema (can be seen as cystic hygroma in utero), low ears, hand/foot edema, hyperconvex nails, a wide chest with valgus at the elbows. Twenty percent of patients have aorta coarctation. Many do not present classically and may be diagnosed in their early teens when stature is short and no sexual maturation ensues. Most have a normal IQ. C. Choice C is incorrect because this describes Patau syndrome due to an extra chromosome 13. Presentation includes a general theme of shifting facial features toward the midline: microphthalmia (some present with a midline Cyclops eye), microcephaly with severe mental retardation, polydactyly, cleft lip and palate, cardiac and renal defects, umbilical hernias, and cutis aplasia (particularly in the midline of the scalp). D. Choice D is correct because an extra chromosome 21 is indicative of Down syndrome. Patients with Down syndrome will present with the features described in the vignette as well as small ears, redundant nuchal skin, clinodactyly (the fifth digit is shorter and curved toward the radius). Additionally, cleft lip or palate, strabismus, and hypothyroidism may be seen. E. Choice E is incorrect because this describes Klinefelter syndrome. These patients are males who have inherited an extra X chromosome. At birth they usually appear unremarkable without findings. Findings are variable but include testicular atrophy with resultant infertility, eunuchoid body (unusually long limbs compared to the body), and gynecomastia. IQ may be normal or in the low-normal range

Mark is a 7-year-old boy who presents to your clinic with recurrent abdominal pain for the last 5 months. His pain was intermittent but progressed over the last 2 weeks. He complains of diffuse abdominal pain that feels like cramps. He has recent onset of diarrhea with gross blood. He is afebrile in the clinic. On physical exam, he is found to have pallor and mild tenderness in the LLQ and RLQ. When you plot his weight and height on his growth chart, you notice he has "fallen off the curve." His labs show a normal WBC count but decreased hemoglobin and hematocrit. Biopsies taken from the terminal ileum during colonoscopy show transmural inflammation. Which of the following is the best first line of treatment for this patient? A Gluten-free diet exclusively B Antibiotics C Surgical resection D 5-aminosalicylic acid E Corticosteroids

D has been selected by the expert. A. A gluten-free diet would be appropriate if patient was diagnosed with celiac disease. The classic presentation of celiac disease is chronic abdominal pain, abdominal distention, diarrhea, anorexia, vomiting, and poor weight gain. Diagnosis can be made with IgA tissue transglutaminase (TTG) antibody or by the finding of villous atrophy on biopsy. B. Antibiotics could be used if this was acute bacterial gastroenteritis such as Shigella or Campylobacter. However, considering the chronic nature of the disease, it is unlikely gastroenteritis. Antibiotics can also be prescribed for Crohn's disease, but they are never the first line of treatment. C. Surgical resection is usually reserved for complications of Crohn's disease, such as obstruction or fistula formation. D. This patient's clinical presentation and biopsy results are consistent with Crohn's disease (CD). You would expect transmural inflammation in Crohn's (as opposed to inflammation limited to mostly mucosa in ulcerative colitis). First-line therapy for CD is 5-aminosalicylic acid. Patients who have incomplete response may be treated with corticosteroids, antibiotics, immunomodulators such as azathiprine, 6-mercaptopurine, or biologics such as Infliximab, a monoclonal antibody to TNF-alpha. E. Corticosteroids can also be used in Crohn's disease but they are not the first line of treatment.

A 4-year-old patient with sickle cell disease presents for a well child evaluation. She has a history of three sickle cell vaso-occlusive crises in the past, including dactylitis and bone pain. She has been symptom-free for a few months and today she is feeling well. She is meeting her developmental milestones. She had an upper respiratory infection recently, but seems to be getting better now. She is up to date on her standard vaccinations up to 2 years including a full course (four doses) of Prevnar. What would you do for her today? A Echocardiogram and transcranial ultrasound B Quantiferon gold test C HPV vaccine D A dose of pneumococcal polysaccharide vaccine (Pneumovax). E Chest x-ray

D has been selected by the expert. A. Echocardiogram and transcranial ultrasound are recommended for patients with sickle cell disease. Echocardiogram screens for evidence of left ventricular hypertrophy and cardiomyopathy. Transcranial ultrasound is used to evaluate risk of stroke by measuring the middle cerebral artery flow. Routine transcranial ultrasound screening for these patients is recommended; echocardiogram is usually recommended for patient older than 21 years old, or for those with cardiopulmonary symptoms. B. A quantiferon gold test assesses previous exposure to TB by measuring for production of IFN by memory T cells from heparinized blood. This test is an acceptable way to measure whether a patient has been infected with TB, including latent infection, but is not indicated in this setting. C. HPV is recommended for children starting at age 11 years old. D. All children should routinely receive the Prevnar vaccine series. Patients with risk factors for pneumococcal sepsis, such as those with sickle cell disease, damaged spleen or asplenia, cochlear implants, CSF leaks, HIV, immunocompromise, chronic hear/lung disease, or those taking immunosuppressive medication should also receive the 23-valent polysaccharide pneumococcal vaccine at age 2 years. E. Chest x-ray is not recommended when there are no current URI symptoms.

8-year-old Jenny presents complaining of intermittent, crampy abdominal pain that has persisted over the last three months. The pain is nonspecific, nonfocal, and not associated with any other systemic symptoms such as fever, chills, weight loss, nausea, vomiting or diarrhea. The pain also seems to occur more frequently during the week and not as often on weekends. The abdominal exam is normal. Jenny is given a diagnosis of functional abdominal pain and scheduled for a one-month follow-up. Six months later, she returns to the clinic complaining of more frequent, more severe abdominal pain that is waking her up at night. She also reports a week of diarrhea containing mucus and blood without associated fever or vomiting. Review of her growth chart demonstrates a slowing of weight gain and a drop in height velocity. What is the most likely diagnosis? A Irritable bowel syndrome B Giardiasis C Celiac disease D Crohn's disease E Henoch-Schonlein purpura (HSP)

D has been selected by the expert. A. Irritable bowel syndrome is a symptom-based diagnosis with chronic abdominal pain, discomfort, bloating, and alteration of bowel habits. It is often a diagnosis of exclusion, with no known organic cause. Other symptoms include diarrhea or constipation. B. Giardiasis is a parasitic disease caused by flagellate protozoan that inhabits the digestive tract leading to decreased appetite, diarrhea, hematuria, loose or watery stool, stomach cramps, bloating, excessive gas, and burping. Many people with giardia infection are asymptomatic. Untreated symptoms may last six weeks or longer. Diagnosis is made via stool microscopy, ELISA, or entero-test with gelatin capsule and thread. C. Celiac disease is an autoimmune disorder of the small intestine characterized by diarrhea, failure to thrive, and fatigue. A rash known as dermatitis herpetiformis (an autoimmune cutaneous eruption) can also occur. D. Crohn's disease is most consistent with this presentation, as it affects GI tract from mouth to anus, leading to abdominal pain, diarrhea (can be bloody), vomiting, or weight loss. Extraintestinal symptoms include skin rashes, arthritis, and fatigue. Fever, fistula, and perianal complications are also common. E. Henoch-Schonlein purpura (HSP) presents with abdominal pain, palpable purpura on the lower extremities, joint pain, and often kidney involvement. It is an acute systemic vasculitis characterized by deposition of immune complexes containing IgA. Abdominal pain is colicky often with nausea, vomiting, constipation, or diarrhea.

A 3-year-old girl presents to the ED with sudden onset difficulty walking. She does not have a fever, headache, nausea, or vision changes, but two weeks ago she had a runny nose, a fever, and a rash. Musculoskeletal exam reveal no abnormalities of lower extremities. Neuro exam reveals bilateral horizontal nystagmus, wide based stance and swaying, and bilateral overreaching on finger to nose test. An LP is performed which reveals a normal CSF. Which of the following is the most likely diagnosis? Single Choice Answer: Please select one answer. A Opsoclonus-myoclonus syndrome B Hydrocephalus C Infectious cerebellitis D Post-infectious cerebellitis E Migraine headache

D has been selected by the expert. A. Opsoclonus-myoclonus syndrome is incorrect. It is a paraneoplastic syndrome that occurs most often with neuroblastoma in a young child who presents with ataxia and jerking or erratic movements as well as jerky conjugate movements of the eyes. B. Hydrocephalus is incorrect because the presentation tends to be more gradual or insidious in onset and it is usually associated with headache and vomiting. C. Infectious cerebellitis is incorrect because it presents with fever and sometimes mental status changes. Pathogens that cause this may include EBV, mumps, and enterovirus. D. Post-infectious cerebellitis is correct. This typically presents in a younger child with ataxia, nystagmus, vomiting and sometimes dysarthria. It is believed to be an autoimmune response leading to demyelination of the cerebellum occurring several weeks after a viral infection such as varicella or coxsackie virus. E. Migraine headache is incorrect because this would present with a headache. Although basilar artery migraines and hemiplegic migraines may present with acute ataxia, headache is an accompanying symptom.

On your first day rotating in the pediatric clinics, you are assigned to see a patient who is 9 weeks old and was brought into clinic by his worried mother. She states that her son has not gained weight since they left the hospital. His weight is < 5th percentile, and height and head circumference are at the 25th percentile. His mother says he drinks two ounces of milk every two to three hours, and suckling is strong without any spitting up during feeds. He poops more than 10 times a day, but it appears greasy and foul smelling. He had an unremarkable birth history and a normal newborn screen. Cardiac, pulmonary, abdominal, and neurologic exams are all normal. His mother mentions her cousin had trouble gaining weight and would get frequent "lung infections." Which of the following is the best next step in management? Single Choice Answer: Please select one answer. A Increase formula to a higher calorie mixture B Stool culture and Wright stain C Swallow study D Sweat chloride test E Echocardiogram

D has been selected by the expert. A. Two ounces of milk every two to three hours is adequate intake. This child may need a higher calorie formula in the future, but not before a diagnosis has been made, as there is likely an organic cause of failure to thrive. B. Stool culture with Wright stain for WBC's could be useful in a patient with diarrhea if you suspect an infectious cause of the diarrhea. This patient however, presents with no fever and no blood in the stool, and a good appetite, all of which are less consistent with an infectious etiology. C. A swallow study would be used to evaluate this child's ability to drink fluids, but since he has not been spitting up and appears to take in milk without difficulty, this test would be less useful at this time. D. The patient's greasy and foul-smelling stool is characteristic of steatorrhea, or fat in the stool. This occurs in patients with cystic fibrosis due to their impaired pancreatic exocrine function. Given the stool history in the face of failure to thrive, a sweat chloride test would be indicated at tis time. Of note, the family history of poor weight gain in a cousin with frequent lung infections also suggests the possibility of cystic fibrosis-related bronchiectasis. Cystic fibrosis should still be suspected in the child who has a normal newborn screen. States differ in the type of test offered to screen for cystic fibrosis, and although great strides have been made in newborn detection, methods are not 100% sensitive. E. An echocardiogram may be useful if you suspect congestive heart failure as the cause of failure to thrive. In this case, however, the cardiac exam was normal and there were no other signs of heart failure, such as tachypnea, tachycardia, hepatomegaly, lower extremity edema, or crackles on lung exam.

In the secondary prevention of ischemic stroke, when using Aspirin alone as anti-platelet therapy, which of the following doses is recommended; and on what basis? A Higher dose; more anti-platelet activity B Lower dose; less bleeding risk C Either high or low dose aspirin; they confer equivalent benefit and similar risk D Low dose aspirin; confers equivalent benefit to high dose, and less bleeding risk

D has been selected by the expert. The correct answer is D. Although in the past it was believed that patients on a 325 mg dose of aspirin compared to those on an 81 mg dose where conferred more protection from ischemic stroke, studies now show that increasing the dose of aspirin, while increasing the risk of hemorrhage, does not confer more benefit than the 81 mg dose. For that reason, the 81 mg dose of aspirin, is sufficient to help prevent a stroke in a patient with a TIA or other risk factors for stroke.

A 20-year-old female who is a long standing patient at this clinic with no significant past medical history presents with first-time onset of dysuria accompanied by frequency and urgency for the past day. She thinks that there is a strange odor to her urine but denies any hematuria. In addition, she feels mild lower abdominal discomfort but denies fevers, chills, nausea, vomiting, constipation, diarrhea, or costo-vertebral angle (CVA) pain. She reports no known allergies She has never been sexually active and has no vaginal discharge nor irritation. LMP was one week ago and was typical. She wonders if using a perfumed bubble bath for the first time a few days ago might have triggered her symptoms. On exam, her vital signs are stable, she has no CVA tenderness and mild suprapubic discomfort. Urinalysis of a mid-stream catch is within normal limits and a pregnancy test on the same sample, performed despite her history, is negative. Which of the following reflects best management in this situation? A Insist that a pelvic exam and cervical DNA probe be performed today B Await urine culture results before any treatment C Reassure that this is not a urinary tract infection D Trimethoprim/sulfamethoxazole one tab twice daily for three days E Urine DNA probe

D has been selected by the expert. There is very good evidence that common typical symptoms of urinary tract infection (UTI) (eg. dysuria and frequency) with the absence of vaginal symptoms are highly predictive of UTI in young women who have no systemic symptoms. The constellation of typical symptoms outweighs a normal urinalysis. Therefore, based on the symptoms presented, the most reasonable treatment option is to treat empirically with a standard antibiotic for a short course. It is not unreasonable to send urine for culture but management does not need to wait till that result is available. While it is always prudent to consider the possibility of sexually transmitted infection in this age group, based on the information presented about an established patient, it would be inappropriate to insist on a pelvic exam or await results of a urine DNA probe in the face of such strong UTI symptoms.

A 22-year-old female with no significant past medical history experienced an inversion type injury to her right ankle while playing volleyball. The ankle quickly became edematous, but she used ice and was able bear weight on the foot. When she comes to clinic two days later, there is mininmal edema, she has good motor function, and has normal sensation. She has tenderness at the anterior lower lateral malleolus but not inferiorly nor posteriorly. X-rays are not indicated. You recommend continued relative rest and also tellthe patient to keep it elevated and ice it several times during the day to help with the pain and swelling. You inform the patient that immobilization and compression is good for the conservative management of her condition. What is the best compression device to use in this situation? A Tape B Compression stockings C Elastic wrap D Semi-rigid ankle support E Solid cast

D has been selected by the expert. This patient apparently has a ligamentous injury to her anterior talo-fibular ligament. In recommending the RICE mnemonic, a semi-rigid ankle support (like an Air Stirrup) provides protection from repeat inversion injury while allowing the patient to actively dorsi- and plantar- flex her foot which aids recovery. Compression stockings and elastic wrap do not provide adequate support. Solid cast completely immobilizes the ankle which delays recovery while the evidence supporting taping of the ankle is lacking.

Luanne is a 15-year-old female who presents with three hours of abdominal pain and two episodes of non-bilious, non-bloody vomiting. She rates her pain at 8/10 and describes it as constant, located mainly in the middle of her belly but somewhat present throughout her abdomen. It is worse with coughing and moving. She has never had pain like this before, and has had no appetite since the pain started. She is sexually active with her boyfriend of three months, always uses condoms, and has not been tested for STIs. She is due to start her period next week. Vitals: 37.9 C, HR 100 bpm, BP 120/85 mmHg, RR 14 bpm. On exam, she exhibits involuntary guarding, mild rebound tenderness, and tenderness to palpation between her right anterior superior iliac spine and umbilicus. On pelvic exam, she reports tenderness when attempting to palpate her right adnexa, but no masses are appreciated and there is no cervical motion tenderness. Her WBC and CRP are within normal limits. Based on the information above, what is the most likely diagnosis? A Ovarian torsion B Pelvic inflammatory disease C Ectopic pregnancy D Appendicitis E Cholecystitis

D has been selected by the expert. D. Appendicitis is the most common condition in children requiring immediate surgical intervention, but often presents differently than in adults (especially in infants). Aspects of their atypical presentation include lack of migration of pain to the RLQ, negative Rovsing's sign, and involuntary guarding and fever without perforation. In school-age children who can articulate the pain, they often describe pain with movement or coughing (cat's eye sign). Also, rebound tenderness was found to be neither sensitive nor specific in the pediatric population, while in the adult population it is one of the most accurate PE findings (86%). Luanne is of the older pediatric population, and so will present with a more typical appendicitis. Her sudden onset of intense pain at the umbilicus with vomiting, anorexia, and tenderness at McBurney's point are all classic findings. The more atypical signs include diffuse pain centered below the umbilicus, and rebound tenderness that might point to a perforation (more likely, it is part of the atypical pediatric presentation given her normal WBC study). Another atypical aspect of her exam is her adnexal pain during the pelvic exam, which could be due to the degree of inflammation and the positioning of her appendix. The key take-away point is to have a high index of suspicion for appendicitis for pediatric patients with abdominal pain given their atypical presentation. A. Ovarian torsion is more common in the post-menopausal population, though it can present in any age group. It is described as intermittent stabbing pain in the lower abdominal or pelvis. Torsion is often secondary to an ovarian mass, such as a neoplasm or corpus luteal cyst, which is occasionally appreciated on exam. Nausea and vomiting are very common findings as well. Ultrasound is essential to initial workup. Given that Luanne has periumbilical pain, tenderness at McBurney's point, and no palpable masses on pelvic exam, ovarian torsion is a less likely diagnosis. B. Pelvic inflammatory disease is definitely a possibility given her sexual history, lack of STI screening, and adnexal pain. However, this pain is often post-coital and also first occurs during or immediately following menstruation. Another key finding is mucopurulent discharge and cervical motion tenderness, both of which are absent in Luanne. RUQ pain and a fever (present in 50% of patient) are other signs not seen in this case. C. Patients with ectopic pregnancy typically present with painless vaginal bleeding six to eight weeks after their last menstrual period. The pain is described as crampy pelvic pain, and it is often associated with nausea. Diffuse abdominal pain is also present if rupture and intraperitoneal bleeding occurs. Given that Luanne is three weeks from her last menstrual period, reports no vaginal bleeding, and her pain is intense and located in the middle of her abdomen, an ectopic pregnancy is less likely. E. The pain of cholecystitis is steady, most often present in the RUQ, and can radiate to the right shoulder. It can worsen with ingestion of fat-rich foods, and often causes anorexia with episodes of nausea and vomiting. It is much less common in children than adults. Given Luanne's presentation, age, and exam, cholecystitis is very low on our differential.

Luanne is a 15-year-old female with 3 hours of abdominal pain and 2 episodes of non-bilious, non-bloody vomiting. She rates her pain at 8/10 and constant, located mainly in the middle of her belly, but is somewhat present throughout her abdomen. It is worse with coughing and moving. She has never had this pain before, and has had no appetite since the pain started. She is sexually active with her boyfriend of 3 months, always uses condoms, and has not been tested for STIs. Her last menstrual period was 2 weeks ago. Vitals: 37.9, HR 100, BP 120/85, RR 14. On exam, she exhibits involuntary guarding, mild rebound tenderness and tenderness to palpation between her right anterior superior iliac spine and umbilicus. On pelvic exam, she reports tenderness when attempting to palpate her right adnexa, but no masses are appreciated and there is no cervical motion tenderness. Her WBC and CRP are within normal limits. Based on the information above, what is the most likely diagnosis? A Ovarian torsion B Pelvic inflammatory disease C Ectopic pregnancy D Appendicitis

D has been selected by the expert. D. Appendicitis is the most common condition in children requiring immediate surgical intervention, but often presents differently than in adults (especially in infants). Aspects of their atypical presentation include lack of migration of pain to the RLQ, negative Rovsing's sign, and involuntary guarding and fever without perforation. In school-age children who can articulate the pain, they often describe pain with movement or coughing (cat's eye sign). Also, rebound tenderness was found to be neither sensitive nor specific in the pediatric population, while in the adult population it is one of the most accurate PE findings (86%). Luanne is of the older pediatric population, and so will present with a more typical appendicitis. Her sudden onset of intense pain at the umbilicus with vomiting, anorexia, and tenderness at McBurney's point are all classic findings. The more atypical signs include diffuse pain centered below the umbilicus, and rebound tenderness that might point to a perforation (more likely, it is part of the atypical pediatric presentation given her normal WBC). Another atypical aspect of her exam is her adnexal pain during the pelvic exam, which could be due to the degree of inflammation and the positioning of her appendix. The key take away point is to have a high index of suspicion for appendicitis for pediatric patients with abdominal pain given their atypical presentation. A. Ovarian torsion is more common in the post-menopausal population, though it can present in any age group. It is described as intermittent stabbing pain in the lower abdominal or pelvis. Torsion is often secondary to an ovarian mass, such as a neoplasm or corpus luteal cyst, which is occasionally appreciated on exam. Nausea and vomiting are very common findings as well. Ultrasound is essential to initial workup. Given Luanne's pain localized around her belly button, her tenderness at McBurney's point, and lack of palpable masses on pelvic exam, ovarian torsion is a less likely diagnosis. B. Pelvic inflammatory disease is definitely a possibility given her sexual history, no STI screening in the past, and adnexal pain. However, this pain is often post-coital and also first occurs during or immediately following menstruation. Another key finding is mucopurulent discharge and cervical motion tenderness, both of which are absent in Luanne. RUQ pain and a fever (present in 50% of patient) are other signs not seen in this case. C. Patients with an ectopic pregnancy typically present with painless vaginal bleeding 6-8 weeks after their last menstrual period. The pain is described as crampy pelvic pain, and it is often associated with nausea. Diffuse abdominal pain is also present if rupture and intraperitoneal bleeding occurs. Given that she is 2 weeks from her last menstrual period, she reports no vaginal bleeding and her pain is intense and located in the middle of her abdomen, an ectopic pregnancy is less likely.

A 3-week-old infant is brought to the pediatrician for failure to thrive (despite adequate, even prolonged, feedings) and respiratory distress (particularly tachypnea). EKG shows high voltage QRS complexes in leads V1 and V2. What other features does this infant most likely have? A Cyanosis from a right-to-left shunt B Systolic murmur with a widely split second heart sound C Continuous murmur that is louder during systole D Left-to-right shunt

D has been selected by the expert. D. Correct. A heart murmur from a VSD is typically not appreciated in the immediate newborn period, as the pulmonary vascular resistance is still quite elevated. During this time, since the pulmonary vascular resistance equals the systemic vascular resistance, there is no shunting of blood through the open VSD. However, after a few days to weeks after birth, the pulmonary vascular resistance decreases, and the murmur appears, reflecting the shunted flow of blood through the open VSD (from left to right). A. Incorrect. After the pulmonary resistance of the newborn period diminishes, ventricular septal defects (VSD) present with a left-to-right shunt. If unrepaired spontaneously or surgically, this ultimately leads to increased pulmonary vascular resistance, termed Eisenmenger's syndrome. When this occurs, the pulmonary vascular resistance equals and then exceeds that of the systemic vascular resistance, leading to reversal of the direction of blood flow through the VSD to become a right-to-left shunt. However, this generally occurs if the VSD is allowed to persist for months to years, which is less likely in our 3-week-old infant. B. Incorrect. A widely split, fixed S2 indicates an atrial septal defect (ASD). These are often detected in children when they're between 3 and 5 years old. The systolic murmur is due to the increased blood flow across the pulmonic valve. The widely split fixed second heart sound indicates an ASD is the cause of the murmur rather than an innocent heart murmur. C. Incorrect. A continuous murmur that is louder during systole describes a patent ductus arteriosus (PDA). A PDA, like a VSD, is also discovered during infancy. However, a holosystolic murmur with a palpable thrill at the left lower sternal border is more characteristic of a VSD. The murmur of a VSD is from the left-to-right shunting of blood during systole due to systemic vascular resistance exceeding pulmonary vascular resistance.

A 4-year-old patient presents with several months of cough. Mom also reports a history of red skin patches, which are pruritic, and allergies to peanuts, eggs, and mangoes. Which of the following would be characteristic of the cough that this patient would present with? A Does not awaken patient from sleep B Paroxysmal C Barking cough D Worse at night E Associated with crackles on exam

D has been selected by the expert. A. This choice is incorrect. This patient has asthma, which commonly presents with symptoms awakening the patient from sleep. Habitual cough disappears at night. B. This choice is incorrect. Paroxysmal coughs are associated with bacterial infections such as pertussis, Chlamydia, or mycoplasma. Foreign bodies can also produce sudden onset of cough. History could help to determine if the latter is a cause. C. This choice is incorrect. Barking coughs are associated with croup or other forms of subglottic disease. Foreign bodies can also produce this type of cough. D. This choice is correct. Asthma frequently presents with nighttime exacerbations. The cough often presents with wheezing and is usually a dry cough. E. This choice is incorrect. Diseases associated with crackles usually have intrinsic pulmonary involvement. Crackles can be fine or coarse and usually represent alveolar or small airway conditions.

A 6-year-old boy presents to the ED with three days of diffuse muscle aches and occasional chills. Today, he had a headache and abdominal pain. He reports that he does not feel hungry because he feels sick to his stomach. He denies recent cough, congestion, sore throat, joint pains, or sick contacts. His vitals are: T 101.3 F, BP 108/71 mmHg, P 110 bpm, R 28 bpm, O2 sat 100% on RA. On physical exam, you notice blanching, erythematous macules on his ankles and several petechiae on his wrists. Upon questioning, his mother says that the spots on his wrists previously looked like the spots on his ankles. His neck is supple and there is no hepatosplenomegaly or lymphadenopathy. He reports no sick contacts, but recently visited his cousins in North Carolina. What is the best next step in management? A Give acetaminophen, obtain a Monospot, write a note for activity restriction, and advise his mother to bring him back if he is unable to tolerate fluids B Perform skin scraping of macules and examine under microscope with KOH prep C Admit the patient, obtain CBC, blood and CSF cultures, and await culture results to guide antibiotic therapy D Admit the patient, obtain CBC, blood and CSF cultures, then give loading doses of doxycycline 2.2 mg/kg and ceftriaxone 100 mg/kg/day E Give acetaminophen and obtain CBC, UA, and BUN/Cr

D has been selected by the expert.D. This choice is correct. Given the patient's abdominal pain, headaches, myalgias, fever, and nausea, followed by blanching erythematous macules, which may be transitioning to petechiae and purpura, this presentation is classic for RMSF. His recent travel to North Carolina also fits with the geographical distribution of RMSF. The treatment of choice is doxycycline. N. meningiditis coverage with ceftriaxone is also necessary given his rash, headaches, and fevers. A. This choice is incorrect. Although infectious mononucleosis is a possibility, this presentation is concerning for Rocky Mountain Spotted Fever (RMSF). Of note is the absence of sore throat, lymphadenopathy, and hepatosplenomegaly, which suggest something other than mononucleosis. B. This choice is incorrect. Scabies or a fungal infection such as tinea versicolor would not explain his fever, headache, abdominal pain, petechiae, and myalgias. For scabies, his chief complaint might be intense itching, and he might have an exposure history. C. This choice is incorrect. Given the history of fever, headache, and petechial rash, you cannot yet rule out meningitis and should give empiric antibiotics immediately. Then, once you have established a diagnosis of RMSF, treatment with doxycycline is indicated. E. This choice is incorrect. Although Henoch-Schonlein Purpura can present with abdominal pain and a rash on the lower extremities, it usually begins with petechiae and purpura rather than a blanching rash. Arthralgias are also absent.

A 12-month-old previously healthy girl presents with cough and mild subcostal retractions. She is afebrile, and physical exam reveals asymmetric wheezing. Chest x-ray demonstrates unilateral air trapping. What is the most likely diagnosis? A Croup B Pneumonia C Acute bronchiolitis D Foreign body aspiration E Asthma

D. Features of foreign body aspiration include unexplained wheezing and asymmetric breath sounds, as well as air trapping in one lung indicating unilateral airway obstruction. The right main bronchus is the more commonly obstructed due to anatomy (it is wider and more vertical than the left). The most commonly aspirated foods are hot dogs, nuts, hard candy, grapes, and popcorn. A. Croup involves subglottic inflammation, typically presenting with inspiratory stridor and a "barky cough" (i.e., like a seal). This patient is not noted to have either, and also presents with asymmetric wheezing and air trapping that would not be expected in an individual with croup. B. While this patient presents with cough and increased work of breathing, she is afebrile, and auscultation of the chest does not reveal crackles or decreased breath sounds/area of consolidation, which would be consistent with pneumonia. Additionally, chest x-ray findings are not consistent with a lobar or more diffuse pneumonia. C. Acute bronchiolitis is a good thought, especially as this is the most common cause of wheezing in infant; however, if this were the diagnosis, the patient would most likely be febrile and chest x-ray would demonstrate scattered atelectasis and/or diffuse opacities from bronchial obstruction. E. While the finding of wheezing is consistent with asthma, this patient has wheezing only on one side. Along those lines, chest x-ray in an asthmatic patient would demonstrate global air trapping with hyperinflated lungs, rather than unilateral findings.

A 2-year-old female with normal birth and developmental history presents with increased agitation and decreased arousability. Her father suffers from chronic pain secondary to a back injury, and her mother found an open container of pills on the bed. Vitals reflect bradycardia, bradypnea, hypotension, and slight hypothermia. On physical exam, she exhibits somnolence, constricted pupils, hypoactive bowel sounds, and hyporeflexia. What substance was most likely ingested? A Iron B Amitriptyline C Insecticides D Hydromorphone

D. Hydromorphone is correct. Opioids such as hydromorphone can cause respiratory depression, bradycardia, hypotension, hypothermia, constipation, nausea, vomiting, sedation, confusion, and/or miosis A. Iron is incorrect. Iron toxicity tends to cause severe abdominal symptoms followed by signs of shock. The child would be at risk for coagulopathies, gastrointestinal hemorrhage, and/or metabolic acidosis. B. Incorrect. Amitriptyline is an anticholinergic medication that belongs to the tricyclic antidepressant family. Anticholinergic toxicity can cause fever, dry, and flushed skin, urinary retention, hypertension, tachycardia, mydriasis, and/or decreased gastric motility. C. Incorrect. Ingestion of insecticides, which typically are organophosphates, can induce increased lacrimation, salivation, sweating, urination, bronchorrhea, bronchospasm, muscle twitching, muscle weakness, bradycardia, miosis and blurred vision, and/or increased gastric motility.

Ms. Brady, a 78-year-old female prevents to your office after six months for follow-up. Her interval history is significant for a TIA three months ago. Today her MMSE is 19. You note that six months ago her MMSE was 22 and nine months ago it was 26. Physical exam shows T=98.8, BP=167/95, HR=76, RR=14. Chest: regular, rate and rhythm, no murmurs, Lungs: clear to auscultation, Neuro: weakness in the right upper extremity, Abdomen: soft, non-tender. She takes atorvastatin and aspirin. Lab studies show Hgb A1c: 5, TSH: 3, B12: 500 pg/mL. Which of the following is the most important recommendation to prevent further disability in this patient? Multiple Choice Answer: A Nothing can be done B Start metformin C Start vegetarian diet D Weight loss E Start hydrochlorothiazide

E has been selected by the expert. The correct answer is E. To prevent additional TIAs and stroke, her blood pressure needs to be controlled. Hydrochlorothiazide is a reasonable first line medication. Her HgbA1C is in the normal range so metformin would not be an appropriate choice. A vegetarian diet would have little impact on limiting dementia, and we don't know what she weighs and whether weight loss would be appropriate.

A 19-year-old female with no significant past medical history is involved as the driver in a motor vehicle accident and brought to the ED by EMS. She is complaining of severe pain in her right lower extremity that has been worsening since the accident. In addition, she has started to notice what she describes as "burning and tingling" in her right foot. On physical exam, her right calf is edematous and tender with tense overlying skin. There is no swelling or tenderness of the right foot or ankle but the right dorsalis pedis and posterior tibial artery pulses are barely palpable. She cannot confirm light touch of the foot and cannot wiggle her toes on command. What is the next best step in the management of this patient? A Reassurance and icepacks q 2 hours B Immobilize leg and ankle with a cast C Urgent EMG of the right lower extremity D Diagnostic imaging of right foot and ankle E Emergent fasciotomy

E has been selected by the expert. This clinical scenario describes acute compartment syndrome which is a vascular emergency. Emergent fasciotomy is the treatment of choice to relieve pressure in the calf and, if not performed, the limb could be lost due to acute ischemia. While emergent radiographs of the tibia and fibula are appropriate to evaluate for co-existent fracture, X-rays of the foot and ankle are not indicated. Reassurance, ice packs, urgent EMG and immobilization are all incorrect treatments and place the patient at risk of serious permanent adverse outcome.

A 1-month-old African-American male presents to your office for a check-up. The baby was born at term by NSVD to a 29-year-old G1P0 mother with no complications. Mother states the baby was feeding well until a week ago, when he developed increased sleepiness, prolonged feeding, and greater duration between feeds. His mother notes he stops to take breaks sometimes because he seems to be trying to catch his breath. He has 4 to 6 wet diapers per day and poopy diapers 3 or 4 times per day. Vital signs are: T: 37.6 C, RR: 68 bpm, P: 138 bpm, BP: 88/58 mmHg, and 02 saturation is 98%. The physical examination is notable for increased respiratory effort and retractions, and, upon cardiac examination, a murmur with a hyperactive precordium and no cyanosis. Abdominal exam reveals a liver edge palpable to 4 cm below the right costal margin. Which condition would be least likely to be the cause of the patient's symptoms? A Aortic stenosis B Coarctation of the aorta C Ventricular septal defect D Patent ductus arteriosus E Atrial septal defect

E has been selected by the expert. E. Choice E is correct because atrial septal defects (ASDs) do not cause CHF. An ASD malformation is a left-to-right shunt, and—depending on the size of the defect—the patient may or may not present with symptoms. ASDs often go undiagnosed for decades due to subtle physical examination findings and/or a lack of appreciable symptoms. If the defect is large enough, pediatric patients may present with easy fatigability, recurrent respiratory infections, or exertional dyspnea A. Choice A is incorrect because aortic stenosis is one of the heart defects that present with a murmur and signs of congestive heart failure in infancy. An estimated 10-15% of patients with aortic valve stenosis present when they are younger than one year of age. Neonates with critical stenosis are typically symptomatic and present with symptoms of congestive heart failure—including poor feeding, rapid breathing, poor urine output, and fussiness—as the ductus arteriosus closes and systemic blood flow decreases. B. Choice B is incorrect because coarctation of the aorta is one of the heart defects that present with a murmur and signs of congestive heart failure in infancy. Pediatric patients may present in the first few weeks of life with poor feeding, tachypnea, and lethargy and progress to overt CHF and shock. Symptoms may be subtle at first, and patients may make repeated trips to the physician before being diagnosed. Presentation after the neonatal period is usually consistent with hypertension or a murmur. Patients usually have not developed overt CHF because of the presence of arterial collateral vessels. C. Choice C is incorrect because ventricular septal defect is one of the heart defects that present with a murmur and signs of congestive heart failure in infancy. Signs and symptoms often appear during the first few days, weeks, or months of a child's life. VSD is an acyanotic congenital heart defect, manifesting as a left-to-right shunt. A holosystolic murmur is often appreciated, with larger VSDs causing a parasternal heave. An infant with a large VSD will fail to thrive and become diaphoretic and tachypneic, especially with feeding. D. Choice D is incorrect because a large patent ducutus arteriosus (PDA) is one of the heart defects that present with a murmur and signs of congestive heart failure in infancy. A PDA is more common in premature infants and those with neonatal respiratory distress syndrome. A smaller PDA may not cause any symptoms, but infants can present with tachypnea, poor feeding, tachycardia, shortness of breath, fatigue, diaphoresis, and poor growth.

You have accepted a part-time tutoring job for first-year medical students. One of your students asks if you would please clarify the details of normal fetal circulation. Which of the following best describes the path of the majority of the blood that enters the right atrium? A RA > foramen ovale > LA > LV > systemic circulation B RA > RV > VSD > LV > systemic circulation C RA > RV > pulmonary circulation > LA > LV > systemic circulation D RA > RV > ductus arteriosus > LV > systemic circulatio E RA > RV > ductus arteriosus > systemic circulation

E has been selected by the expert. E. Correct. The majority of the fetal circulation travels this route. Approximately 90-92% of the blood that enters the RV (two-thirds of the blood that enters the RA) travels out and through the ductus arteriosus, bypassing the pulmonary circulation and the left heart, ending up in the descending aorta. This blood is perferentially less oxygenated than that which flows through the foramen ovale. Like the foramen ovale, closure of this bypass is a normal transition from intra to extrauterine life. A. Incorrect. In fetal circulation, the foramen ovale connects the RA to the LA, allowing a portion of the blood to bypass the RV and the lungs. Approximately a third of the blood that enters the RA passes through this route (preferentially the most oxygenated which is then delivered to the brain and heart), leaving the majority of the blood to travel into the RV. Closure of the foramen ovale is a normal transition from fetal to extrauterine circulation. B. Incorrect. VSDs are common congenital heart defects, and are not considered a part of normal fetal circulation. They are commonly associated with other conditions such as Trisomy 13, 18, 21, and maternal SSRI use. C. Incorrect. In utero, without ventilation, the pulmonary vasculature is a high-resistance system. As such, only 8 to 10% of the blood that enters the RV flows through the circulation. D. Incorrect. The ductus arteriosus does not empty into the LV, but rather into the descending aorta.

A 4-year-old girl with a history of type 1 diabetes mellitus was admitted to a local hospital for treatment of DKA. A few hours after the treatment, she develops grunting, tachypnea, and has vomited twice. On exam, her left eye is pointing downward and out on straight gaze. Her diastolic blood pressure is 90 mmHg. What is a likely diagnosis? A Hypoglycemia B Hypokalemia C Hyponatremia D Pneumonia with possible sepsis E Cerebral edema

E has been selected by the expert. E. This choice is correct. Administration of bicarbonate during DKA treatment increases the risk of cerebral edema. Although symptomatic cerebral edema is rare (less than 1%), it is associated with a high mortality rate (over 20%). The signs of cerebral edema are described in the vignette, and include tachypnea, headache, vomiting, third nerve palsy, and high blood pressure. A. This choice is incorrect because although hypoglycemia can be a complication of DKA treatment, it would be unlikely to cause a cranial nerve palsy. B. This choice is incorrect. Although hypokalemia can be a complication of DKA treatment, it would likely present as an elevated BP, muscular weakness or myalgia, as well as muscle cramps, constipation, and hyporeflexia in severe cases, rather than with the symptoms described in this vignette. C. This choice is incorrect because, while hyponatremia can occur (due to the dilutional effect caused by water shifting from the intracellular to the extracellular compartment because of hyperglycemia and increased plasma osmolarity), it would be corrected with the DKA treatment and would not present with a cranial nerve palsy. D. This choice is incorrect because pneumonia would not present with a cranial nerve palsy, although it might be important to evaluate patients with DKA for signs of intercurrent illness, including pneumonia, UTI, and perinephric abscess.

Joe, a previously healthy 11-month-old male with 5-day history of a "cold," is brought to the ED by mom for one day of acute worsening cough and intermittent wheezing. Per mom, the cough was initially dry but has become more "phlegmy," making it difficult for Joe to breathe, particularly when he is feeding or more active. His immunizations are up to date, and he has no known allergies. His family history is significant for a 6-year old sister who was diagnosed with asthma four years ago. On exam, Joe is afebrile, mildly tachypneic with normal O2 saturation. He has prominent nasal flaring and mild subcostal retractions. He has clear rhinorrhea but no evidence of oropharyngeal erythema. Lung exam reveals decreased breath sounds and wheezes on the right. What is the most likely diagnosis? A RSV bronchiolitis B Epiglottitis C Viral URI D Asthma E Foreign body aspiration

E. Given Joe's age, foreign body aspiration should always be included in the differential diagnosis for acute onset wheezing. The lung findings of asymmetric breath sounds and wheezing support this diagnosis. Foreign body in the airway can be confirmed by bilateral decubitus or inspiratory/expiratory chest films, characterized by decreased deflation on the affected side. If complete obstruction, x-ray will generally reveal atelectasis (whiting out) and signs of volume loss (mediastinal shift towards affected side to compensate for loss of volume). A. Bronchiolitis is a lower respiratory tract infection most commonly caused by RSV, which is characterized by bronchiolar obstruction secondary to mucus plugging, cellular debris, and edema. Patients generally present with fever and URI symptoms which progress to a worsening cough, wheezing and shortness of breath. Although this patient does have wheezing, the unilateral wheezing with decreased breath sounds is not consistent with bronchiolitis. B. Epiglottitis was commonly caused by Haemophilus influenzae type B, but can also be caused by Staph and Strep species. Patients may present with fever, dysphagia, drooling, stridor and significant respiratory distress. Patients are generally seen sitting, leaning forward with the neck hyperextended. Epiglottitis has become less common due to immunization with Hib. This diagnosis is less likely in our patient, since his immunizations are up to date, he is afebrile and not in severe respiratory distress. C. Our patient probably developed a viral URI five days ago. An upper respiratory tract infection in children can manifest as fever, rhinorrhea, cough, sore throat and myalgias, and may be accompanied by wheezing. However, our patient's ausculatation findings cannot be explained solely by a viral URI. D. Asthma is caused by inflammation of airway mucosa, mucus hypersecretion, mucosal edema and reversible bronchoconstriction. It generally presents as cough, wheezing, tachypnea and dyspnea worsened by cold air, exercise, allergies and URIs. The mainstay of treatment involves bronchodilators (beta-2 agonists) and inhaled steroids. Asthma is a possible diagnosis in Joe given the family history of asthma; however, it is less likely since he was previously healthy with no history of recurrent cough or wheezing. Furthermore, asthma does not generally present with focal wheezing as heard on Joe's lung exam.

Mr. Turner is a 60-year-old male with a past medical history of hypertension and diabetes who presents with fatigue. Upon further questioning, you realize that Mr. Turner does not have trouble falling asleep but has difficulty staying asleep because he often wakes up short of breath. He also notes that he has swelling in his ankles and feet. He denies having any chest pain or palpitations. On exam, you find that his heart has a regular rate and rhythm. You observe hepatojugular reflux and notice 2+ pitting edema in his legs bilaterally. What is the pathophysiology of his condition? A Death of the myocardial cells due to lack of oxygen leading to dysfunction of the heart B Decreased compliance of the ventricles leading to dysfunctional filling of the heart C Decreased forward flow of blood from the heart due to dysfunction of the ventricles D Dysfunction of the pacemaker cells of the atrium

The correct answer is B. In CHF, decreased compliance of the ventricles leads to increased filling pressures, which leads to congestion. Answer A is describing the pathophysiology of a myocardial infarction, which may lead to CHF, but is not always the cause. Answer C is describing a situation (perhaps an arrhythmia) in which ventricular dysfunction causes abnormal flow of blood from the heart. Answer D is also describing a situation that would lead to an arrhythmia.


संबंधित स्टडी सेट्स

CH 14 (THE BRAIN AND CRANIAL NERVES)

View Set

*Area of Triangles, Classify Triangles & Quadrilaterals, 10.1. Area of Parallelograms, Area of Triangle, Rectangles, and Parallelograms

View Set

CS4345 Software Engineering Priciples Test 1

View Set

Howard University Biology 101 Final Study Guide (Ch.1 - Ch.8)

View Set

AP Human Geography Key Issue 11.3 Part 1 Terms

View Set

Graphic Designer Interview Questions

View Set

Chapter 5: Performance Measurement

View Set

Security+ Assessment Exam 1 (DG)

View Set